Vous êtes sur la page 1sur 141

Work Smart

Time taken: 00:23

Theme:Overdose
A

Acetylcysteine

Desferrioxamine

Dimercaprol

Ethanol

Flumazenil

Glucagon

Naloxone

Observation

Pralidoxime

Penicillamine

Sodium nitrite

From the above list of antidotes select the agent that would be most appropriate in the
following cases:

A 70-year-old farmer is admitted acutely after ingesting an unknown overdose. Examination


reveals a particularly anxious male who is sweaty and salivating. His temperature is 40C and he
has a blood pressure of 90/60 mmHg with a pulse of 65 beats per minute.

Incorrect - The correct answer is Pralidoxime


The first case has features of organophosphate poisoning as suggested by the
hypersalivation, sweating and relative bradycardia indicating increased cholinergic activity.
This should be treated with Pralidoxime.

A 16-year-old girl is admitted after taking a paracetamol overdose with alcohol 4 hours
previously. Her plasma paracetamol concentration is just below the nonagram concentration that
would suggest treatment. Her plasma alcohol concentration is 120 mg/l

Correct
The second case has paracetamol poisoning and although the paracetamol concentration is
below the level to treat on the nonagram this person has consumed large amounts of alcohol
and so should be treated with acetylcysteine.

A 52-year-old vagrant attends casualty with hyperventilation and vomiting. He confesses to


having drunk methanol.

Incorrect - The correct answer is Ethanol


The third case has consumed methanol which may cause blindness, lactic acidosis and liver
failure. Problems are a result of the build up of toxic metabolites of methanol which may be
inhibited by the administration of ethanol.

A 6-year-old child is admitted after consuming her mothers ferrous sulphate tablets. She has had
one haemetemesis and the iron concentration is excessive.

Incorrect - The correct answer is Desferrioxamine


The fourth case has consumed a large amount of ferrous sulphate which needs to be treated
with desferrioxamine.

A 50-year-old female is admitted unconscious after taking an overdose of an unknown


substance. The only history is from her husband who states that she has been depressed and
anxious of late and has been prescribed some medication by the GP. Examination reveals a
Galsgow Coma Scale of 10/15 and she responds and opens her eyes to pain. She has a pulse of
62 beats per minute regular, a blood pressure of 130/80 mmHg and a respiratory rate of
20/minute with a saturation of 96 per cent. The pupils are of normal size.

Incorrect - The correct answer is Observation


The final case has a benzodiazepine overdose overdose with stable observations. She does
not require any Flumazenil and can be observed.

Work Smart
Time taken: 00:32

Theme:Substance abuse
A

Aspirin

Barbiturates

Benzodiazepines

Cannabis

Cocaine

Ecstasy

Hallucinogenic mushrooms

Methanol

Opiates

Solvent abuse

Tricyclic antidepressants

From the given list select the drug that is most likely to be responsible for the presentation of
the following cases:

An 18-year-old female is brought to casualty after collapsing in a night club. Her friends state that
she has taken unknown substances during the night and has been hyperactive. She is
hallucinating and has a Glasgow coma scale of 15. Her temperature is 38.5C, she appears
dehydrated, she has a pulse of 110 beats per minute and a blood pressure of 110/70 mmHg.
Respiratory rate is 22/minute and she has saturations of 99%.

Incorrect - The correct answer is Ecstasy


This young girl has been out clubbing and presents with hyperactivity, dehydration together
with generally non-specific signs but slight hypertension suggests amphetamine use. This is
most likely to be ecstasy - MDMA. Ecstasy may also cause arrhythmias and seizures and
has been connected with some fatalities associated with water intoxication and acute
hyponatraemia.

A 33-year-old female is brought to casualty unconscious. Examination reveals a Glasgow coma


scale of 6, a blood pressure of 120/70 mmHg, a pulse of 52 beats per minute a respiratory rate of
10 per minute with saturations of 85 percent. She has small pupils.

Incorrect - The correct answer is Opiates


The features are of respiratory depression and pin-point pupils which are suggestive of
opiates.

A 26-year-old female presents to casualty in distress. She is agitated and has had a
haematemesis. Examination reveals a temperature of 40C, a pulse of 120 beats per minute and
a blood pressure of 110/80 mmHg. She has a respiratory rate of 38/minute and has saturations
of 100%. Her pupils are normal in size.

Correct
This case has hyperventilation, a pyrexia and has had a haematemesis suggestive of a
gastric irritant - aspirin. This causes a metabolic acidosis with hyperpyrexia in overdose.
Haematemesis due to gastric irritation is a feature and coagulation may be deranged.

A 42-year-old female presents unconscious. She has a Glasgow coma scale of 7, a temperature
of 37.5C, a pulse of 134 beats per minute, a blood pressure of 130/60 mmHg and a respiratory
rate of 22 with saturations of 95%. Examination of the pupils reveals dilated pupils. A bladder is
palpable on examination of the abdomen.

Incorrect - The correct answer is Tricyclic antidepressants


This case has reduced conscious level, irritability, a tachycardia, urinary retention and dilated
pupils. These features suggest an anticholinergic toxicity and from the above list, tricyclic
antidepressants fit. Fits and ventricular arrhythmias are other features.

A 17-year-old male is brought to casualty after being found collapsed in the street. Examination
reveals a Glasgow coma scale of 7, a temperature of 36.5C, a blood pressure of 145/85 mmHg
with a pulse of 70 beats per minute. His pupil size is normal and he has a respiratory rate of 15
with saturations of 96%.

Incorrect - The correct answer is Benzodiazepines


This patient to all intents and purposes is unrousable and asleep. This is most likely to be
due to benzodiazepines.

Work Smart
Time taken: 00:39

Theme:Complications of fractures
A

Avascular necrosis

Compartment syndrome

Fat emboli

Gangrene

Haemorrhagic shock

Malunion

Osteoarthritis

Osteomyelitis

Rhabdomyolysis

Tetraparesis

Venous thromboembolism

From the given list select the most likely complication that accounts for each of the following
cases:
A 26-year-old male receives lower leg injuries playing rugby. On examination, he has a pulse of
120 bpm, a blood pressure of 90/60 mmHg and he has a compound fracture of both tibias.

Incorrect - The correct answer is Haemorrhagic shock


This case has features of shock with tachycardia and hypotension a consequence of large quantities
of blood loss associated with the fracture.

A 24-year-old male sustains a forearm injury but fails to attend for medical attention until the
following day when he presents with forearm pain, swelling and an inability to flex the wrist.

Incorrect - The correct answer is Compartment syndrome


The patient has developed a forearm compartment syndrome as a consequence of swelling and
increased pressure of the muscles within the fascial compartment.

A 40-year-old male receives a compound fracture of his left tibia after falling from a ladder. He
undergoes internal fixation. However, several months after discharge from hospital, the patient is
aware of a breakdown of skin overlying the tibia and a persistent discharge.

Incorrect - The correct answer is Osteomyelitis


This case has a discharging sinus months after a compound fracture. This would suggest an
underlying osteomyelitis.

A 33-year-old male injures his hand whilst playing rugby. He attends casualty but leaves after
waiting two hours without obtaining medical advice. He represents three months later with pain at
the base of the thumb and painful movements of the thumb.

Correct
The patient has fractured his scaphoid and has developed avascular necrosis of the scaphoid.

A 55-year-old male is admitted after falling from his horse. Examination and investigations
confirm a fractured pelvis. Hours later he becomes dyspnoeic, with a pulse of 120 beats per
minute and a blood pressure of 100/70 mmHg and a fall in oxygen saturation to 86%. He is
aware of small petechiae.

Incorrect - The correct answer is Fat emboli


This case has developed dyspnoea hours after sustaining a pelvic fracture. This is too early for
thromboembolism but, in the presence of the petechiae, would suggest fat embolism.

Work Smart
Time taken: 00:44

Theme:Treatment of cardiological patients


A

Aspirin

DC cardioversion

Digoxin

Insertion of chest drain

Intravenous morphine

Intravenous naloxone

Intravenous verapamil

Oxygen therapy only

Pericardial drainage

Pulmonary embolectomy

Warfarin

From the given list, which is the most appropriate treatment for each of the following
patients?
A 72-year-old man with ischaemic heart disease complains of feeling faint for the past hour. He is
pale, sweaty and hypotensive. His ECG shows a regular tachycardia of 180 beats/min with QRS
duration 0.20 secs.

Incorrect - The correct answer is DC cardioversion


This patient has underlying IHD with a regular tachycardia of 180 and shows evidence of distress
this suggests VT and the most appropriate treatment would be DC cardioversion.

A 64-year-old woman with known atrial fibrillation treated with digoxin attends your surgery
complaining of transient loss of vision in the left eye which recovered spontaneously.

Incorrect - The correct answer is Warfarin


This patient has had amaurosis fugax and requires anticoagulation in the form of warfarin.

A 73-year-old man with known carcinoma of the bronchus becomes increasingly short of breath
over the past few days. The chest x ray shows an enlarged heart shadow but no pulmonary
oedema.

Incorrect - The correct answer is Pericardial drainage


This patient has a bronchial carcinoma with a pericardial effusion. Pericardiocentesis would be most
appropriate.

A 23-year-old man is seen in the Emergency department with 20% pneumothorax of the right
lung. His blood pressure and pulse are stable.

Incorrect - The correct answer is Oxygen therapy only


This patient has a stable pneumothorax and observation with oxygen therapy is all that is required.

A 75-year-old woman with chronic obstructive pulmonary disease is brought to the Emergency
department, semi-conscious and cyanosed. One week ago she was given a new drug relief for
symptoms of osteoarthritis. She has bilateral pin-point pupils.

Incorrect - The correct answer is Intravenous naloxone


The history of this patient suggests opiate use and naloxone would be appropriate.

Work Smart
Time taken: 00:50

Theme:Advanced Life Support


A

Adenosine

Amiodarone

Basic life support with adrenaline

Call for the cardiac arrest team

Check for a central pulse

Chest tube with an under water seal

Defibrillation at 200J

Give a praecordial thump

Lidocaine

Some other specific treatment if the cause of the arrest is known

Synchronised DC cardioversion

Select the most appropriate answer from the given list that describes the most appropriate
treatment priority or action in the following scenarios:

A 26-year-old known asthmatic has a narrow complex tachycardia with a heart rate of 220/min.
His BP is 85/50 mmHg. Intravenous access is established and high flow oxygen is being given by
mask. Vagal manoeuvres are unsuccessful.

Incorrect - The correct answer is Synchronised DC cardioversion


The first patient is in cardiac arrest and in order to activate the chain of survival, calling the cardiac
arrest team is essential. When limited assistance is available, performing effective basic life support
takes second priority over alerting the arrest team.

A 64-year-old female who 10 days ago had a total hip replacement is found unconscious in the
ward toilet. She is unresponsive, apnoeic and pulseless.

Incorrect - The correct answer is Call for the cardiac arrest team

A 51-year-old male is in refractory fine VF. He has received defibrillatory shocks and 1 mg of
adrenaline intravenously. One minute of CPR is ongoing.

Incorrect - The correct answer is Amiodarone


Amiodarone should be considered in shock refractory eMedicine Refractory VF or pulseless VT. It can
be given as early as the before the fourth defibrillatory shock. Lidocaine can be used as an alternative
when amiodarone is unavailable.

The third case has a peri-arrest rhythm, which is compromising his cardiac output. He needs
to be sedated or anaesthetised prior to being given a synchronised DC shock starting at
100J. Adenosine can induce bronchospasm and should be avoided.

An elderly female has arrested during the insertion of a right subclavian central line. The ECG
rhythm shows sinus tachycardia. Adrenaline has been given and three minutes of basic life
support is ongoing. As the cardiac arrest team leader you notice that the trachea is deviated to
the left.

Incorrect - The correct answer is Some other specific treatment if the cause of the arrest is
known
The fourth case is also in cardiac arrest but has sinus tachycardia as a rhythm, thus it is called PEA
(pulseless electrical activity). Previously it was known as EMD (electromechanichal dissociation). The
deviated trachea suggests a right tension pneumothorax, which requires immediate needle
thoracocentesis to relieve the pressure. A chest tube with an under water seal can be inserted later.

A praecordial thump has just been given to a patient who had a witnessed and monitored VF
cardiac arrest on the coronary care unit. A change in rhythm to a ventricular tachycardia is
observed.

Incorrect - The correct answer is Check for a central pulse


In the fifth case a pulse check is indicated because of the rhythm change, but also because the new
rhythm could also be pulseless VT, which should then be shocked at 200J.

Work Smart
Time taken: 00:58

Theme:Day case anaesthesia


A

Atracurium

Diamorphine

Diazepam

Diclofenac

Etomidate

Fentanyl

Pancuronium

Propofol

Suxamethonium

Thiopentone

From the given list select the most appropriate choice of drug or action in the following
scenarios:
A healthy female is scheduled for a laparoscopic sterilisation. Suitable muscle relaxation will be
provided by which drug?

Correct
Muscle relaxation is required during the creation of a pneumoperitoneum, which provides a
clear view and access to the fallopian tubes. The anticipated duration of surgery is 20 to 30
minutes, thus atracurium is the best choice. Suxamethonium is too short acting and
pancuronium lasts too long.

A young male is due to have a large lipoma excised from his forearm. Which agent would be a
suitable intravenous induction agent to allow placement of an LMA?

Incorrect - The correct answer is Propofol


Propofol is the best induction agent when planning to use an LMA. It inhibits the pharyngeal
and laryngeal reflexes faster than thiopentone or etomidate, providing optimal conditions for
inserting the device.

A well controlled asthmatic patient presents for a knee arthroscopy. Which agent would provide
appropriate intraoperative analgesia?

Incorrect - The correct answer is Fentanyl


Drugs that cause bronchoconstriction or that have the potential to release histamine are best
avoided in asthmatics. Therefore diamorphine and diclofenac are not the best choice.
Fentanyl does not release histamine and small doses will provide suitable intraoperative
analgesia.

A 64-year-old male presents for a cystoscopy. A hiatus hernia was diagnosed six months ago.
His current medication is Gaviscon and he has been nil by mouth since midnight. Select a
suitable muscle relaxant to allow placement of the endotracheal tube.

Incorrect - The correct answer is Suxamethonium


Patients with a hiatus hernia must have their airway protected as quickly as possible
following induction of anaesthesia. Failure to protect the airway with a cuffed endotracheal
tube may lead to soiling of the trachea from regurgitation of residual gastric fluid. Thus a
rapid sequence induction should be considered. Suxamethonium is the correct choice as it
provides optimum intubating conditions in 30 to 45 seconds.

A 21-year-old-male presents for dental extractions under general anaesthesia. He has epilepsy
but has been seizure free for nine months on his current medication. Select an appropriate
intravenous induction agent.

Incorrect - The correct answer is Thiopentone


Thiopentone is a barbiturate and has anticonvulsant properties thus it the correct answer.
Diazepam is an anticonvulsant but it is not used to induce anaesthesia. Propofol and
etomidate have both been known to cause movement resembling convulsions thus are best
avoided.

Work Smart
Time taken: 01:03

Theme:Pre-operative investigations
A

Arterial blood gases

Chest x ray

ECG (12 lead)

Echocardiograph

Full blood count

Glucose concentration

Haemoglobin A1c concentration

Haemoglobin electrophoresis

Lung function test

Prothrombin time and activated partial thromboblastin time

Urea and electrolytes

Select the most appropriate investigation from the given list for the following scenarios:

A 16-year-old African male presents with a 2-day-old penetrating wound to his calf. His main
complaint is pain. Pulse and blood pressure are normal and a full blood count reveals a
haemoglobin of 8.5 g /dl. He is booked for exploration and debridement of the wound.

Incorrect - The correct answer is Haemoglobin electrophoresis


This needs to be investigated as a sickling crisis can be precipitated by surgery.

A 63-year-old female is scheduled for a sigmoid colectomy. Twelve months ago she had an
anterior myocardial infarct but has made a good recovery. She can climb a flight of stairs slowly
but gets short of breath. Medication includes an ACE inhibitor. Your examination reveals a loud
systolic murmur, breath sounds are vesicular.

Incorrect - The correct answer is Echocardiograph


She is undergoing a sizeable procedure and needs appropriate assessment of her left
ventricular function.

A 59-year-old previously healthy male, presented 12 hours ago with bowel obstruction and has
become increasingly confused. He is receiving oxygen by mask. The pulse oximeter reads
100%, pulse is 110/min and blood pressure 135/85 mmHg. Intravenous fluid resuscitation is with
5% dextrose, infusing at a rate of 150 ml/hour.

Incorrect - The correct answer is Urea and electrolytes


This patient's confusion in the context of his obstruction suggests marked dehydration with
sepsis and probable renal impairment.

A 33-year-old female has been on warfarin for three months following a pulmonary embolism.
She is scheduled for open reduction and internal fixation of an ankle fracture, sustained whilst
jogging. Her ankle is painful but otherwise she is symptom free and takes no other medication.

Incorrect - The correct answer is Prothrombin time and activated partial thromboblastin time
She requires appropriate assessment of her INR before any operative procedure.

A 19-year-old female with diabetes who is insulin dependent has a penetrating injury to her left
eye. She is on twice daily insulin and had lunch four hours ago. Her operation is expected to
begin within the hour.

Incorrect - The correct answer is Glucose concentration


Glucose always needs to be checked in a diabetic patient and this young woman will require
a sliding scale insulin regime for her operation.

Surgery on anticoagulated patients can result in excessive intraoperative blood loss and
complicate recovery due to haematoma formation.
For elective surgery warfarin can be stopped and surgery postponed until the INR is less
than 1.5.
Intravenous heparin should be substituted for the warfarin if anticoagulation is essential.
Fresh frozen plasma will allow rapid correction of the effect of warfarin and may be
preferable to vitamin K.
In the case of the 33-year-old female her jogging activity suggests that the pulmonary
embolism has not significantly compromised her gas exchange and so blood gas analysis
would not have a high priority.

Work Smart
Time taken: 01:10

Theme:Stroke syndromes
A

Cerebral abscess

Cerebral embolism

Cerebral infarction

Cerebral oedema

Cerebral thrombosis

Intracerebral haemorrhage

Subarachnoid haemorrhage

Subdural haematoma

Transient ischaemic attack

For each of the following patients below choose the single most likely diagnosis from the
given list of options.
Each option may be used once, more than once or not at all.
A 50-year-old woman with thyrotoxicosis and thyroid heart disease complained of palpitations.
She was found to have atrial fibrillation and started on anticoagulants. The next day she
complained of difficulty in moving her left arm.

Incorrect - The correct answer is Cerebral embolism


Embolisation is a risk associated with dysrhythmias, the commonest being atrial fibrillation.
The risk is greatest in the elderly, particularly those with valvular heart disease, diabetes,
hypertension or previous stokes.

A 31-year-old male loses consciousness briefly and is brought into the emergency ward. He
complains of severe headaches and photophobia.

Incorrect - The correct answer is Subarachnoid haemorrhage


Subarachnoid haemorrhage is caused by rupture of an aneurysm and the initial headache or
coma is caused by the sudden rise in intracranial pressure.

A 38-year-old man has been complaining of headaches, dizziness and poor concentration for
some time. He is brought into hospital with weakness on the left side. His haemoglobin is 20g/dl.

Incorrect - The correct answer is Cerebral thrombosis


Polycythaemia vera causes focal neurological signs as a result of thrombosis from increased
viscosity. There is a paradoxical risk of haemorrhage.

A 62-year-old male is found drunk outside a pub in the early hours of the morning. He is
conscious but unable to move the right side of his body. He starts to have partial seizures in
hospital.

Incorrect - The correct answer is Subdural haematoma


Chronic subdural haematomas sometimes present as strokes. A history of previous trauma
to the head or alcoholic abuse is useful. Alcoholics sometimes have suffered trauma in the
past of which they have no recollection.

A 60-year-old hypertensive man loses consciousness. An urgent head CT scan shows a large
hyperdense lesion in the left fronto-parietal lobe.

Incorrect - The correct answer is Intracerebral haemorrhage


Clinically it can be difficult to differentiate between haemorrhages and infarcts. Recent
haemorrhage is hyperdense and therefore brighter than brain tissue on CT scan.

Work Smart
Time taken: 01:16

Theme:Signs of life
A

Barbiturate coma

Brain stem death

Cardiac arrest with agonal rhythm

Clinical diagnosis of death

Hypothermia

Hypoxic cerebral depression

Pulseless electrical activity

Rigor mortis

Sudden cardiac arrest

Ventricular tachycardia

You are called to a cardiac arrest on the medical admissions unit.

For each of the given situations, indicate the most appropriate diagnosis from the list of
options:
A patient on a medical ward is known to have carcinomatosis; there are no signs of life.

Incorrect - The correct answer is Clinical diagnosis of death

A patient has been pulled out of a lake, and has a deep body temperature of 28.1C; the femoral
pulses are not palpable.

Incorrect - The correct answer is Hypothermia

A patient has been found in bed with a deep body temperature of 22.1C; the arms and legs are
stiff.

Incorrect - The correct answer is Rigor mortis

A patient has been in cardiac arrest for 15 minutes; the ECG appears to show a relatively normal
sinus rhythm.

Incorrect - The correct answer is Pulseless electrical activity

A patient has been in cardiac arrest for 10 minutes; the ECG shows wide regular complexes at a
fast rate.

Incorrect - The correct answer is Ventricular tachycardia

The patient with carcinomatosis, when all other options to this question are considered is
most likely to have a clinical diagnosis of death.

Drowning is associated with hypothermia and the femoral pulses are not palpable due to
pulseless electrical activity.
A patient found with no vital signs of life with stiff limbs, that is, rigor mortis, is likely to have
been dead for some considerable time.
When a relatively normal rhythm is present on the monitor in a cardiac arrest situation,
pulseless electrical activity must be considered.
The most likely cause of wide, regular complexes in a patient in cardiac arrest is ventricular
tachycardia. Other causes of wide complexes in this situation could be hyperkalaemia,
bundle branch block (for example, in presence of myocardial infarction) or ST elevation
giving the appearance of widened QRS complex.

Work Smart
Time taken: 01:22

Theme:Oxygen therapy
A

24% oxygen

24% oxygen and antibiotics

24% oxygen and intravenous furosemide

24% oxygen and nebulised bronchodilators

24% oxygen, antibiotics and nebulised bronchodilators

24% oxygen, diamorphine and intravenous furosemide

60% oxygen

60% oxygen and antibiotics

60% oxygen and intravenous furosemide

60% oxygen and nebulised bronchodilators

60% oxygen, antibiotics and nebulised bronchodilators

60% oxygen, diamorphine and intravenous furosemide

For each of the following clinical situations, select from the given list the most appropriate
option for immediate management:
An 18-year-old woman with a previous history of asthma develops acute dyspnoea following a
row with her boyfriend. On examination she is tachypnoeic and distressed, with widespread
wheezes.

Incorrect - The correct answer is 60% oxygen and nebulised bronchodilators


This young woman has severe asthma and needs nebulisers and high concentration of
oxygen.

A previously healthy 24-year-old man presents with acute dyspnoea, left sided and and cough
productive of green sputum tinged with blood. On examination he is pyrexial and looks ill, with
signs of consolidation at the left lung base but no wheezes.

Incorrect - The correct answer is 60% oxygen and antibiotics


This is a young man with a lobar pneumonia with systemic sepsis. He should receive high
flow oxygen and intravenous antibiotics.

A 68-year-old smoker presents with acute shortness of breath, on a background of chronic


productive cough. On examination he is centrally cyanosed and pyrexial, with widespread
crackles and wheezes.

Incorrect - The correct answer is 24% oxygen, antibiotics and nebulised bronchodilators
The history suggests chronic bronchitis with acute exacerbation and chronic CO2 retention
should be suspected. This man may have type 2 respiratory failure with CO2 retention. High
flow oxygen may suppress his respiratory drive. Nebulisers and antibiotics should be
standard given the pyrexia and chest signs.

A 55-year-old man develops sudden onset of right sided chest pain and dyspnoea in Terminal 2
of Manchester Airport on arrival from Hong Kong. On examination he is ill, cyanosed and
apyrexial. There are no localising signs on chest examination.

Incorrect - The correct answer is 60% oxygen

The history is suggestive of pulmonary embolism. The immediate management would be


high flow O2. Heparin should be commenced whilst a definitive diagnosis is being sought.

A 57-year-old man is on the waiting list for coronary artery bypass grafting; he develops acute
dyspnoea and cough with frothy pink sputum. On examination he is tachypnoeic and distressed,
with profuse basal crackles but no wheeze.

Incorrect - The correct answer is 60% oxygen, diamorphine and intravenous furosemide
Acute asthma requires high dose O2 and bronchodilators, along with IV hydrocortisone. The
history of coronary artery disease, and presentation with pink frothy sputum, would suggest
left ventricular failure and pulmonary oedema. Diamorphine, IV diuretic and high dose O2 is
the immediate treatment. IV nitrates (or sublingual GTN) are also an option.

Work Smart
Time taken: 01:29

Theme:Back pain
A

Abdominal aortic aneurysm

Achalasia of the oesophagus

Acute prolapsed intervertebral disc

Ankylosing spondylitis

Chronic low back pain

Chronic pancreatitis

Costochondritis

Dissection of thoracic aorta

Metastatic prostatic carcinoma

Multiple myeloma

Myelomeningocele

Neurofibroma

Osteomalacia

Osteoporotic vertebral collapse

Paget s disease of bone

Pleural effusion

Psoriatic arthropathy

Staghorn calculi

Syringomyelia

Tuberculosis

Uric acid arthropathy

The following patients present to the casualty department with back pain.
What is the most likely diagnosis from the given list of options?
Normal values:
Serum corrected calcium

2.15 - 2.65 mmol/L

Serum creatinine

60 - 110 mol/L

A previously healthy 75-year-old white woman presents with acute onset of severe pain over the
mid-thoracic spine. Examination reveals localised tenderness over T5. Serum calcium and
alkaline phosphatase are normal, and ESR is 20 mm in the first hour.

Incorrect - The correct answer is Osteoporotic vertebral collapse


The most likely diagnosis is an osteoporotic vertebral collapse in a female of this age with a
normal calcium and alkaline phosphatase, with only a mildly raised ESR, which may be
normal for this age group.

A 30-year-old Asian woman complains of poorly localised pain in the shoulders and pelvis.
Examination reveals no localising signs. Serum corrected calcium is 2.1 mmol/l, the alkaline
phosphatase is raised, and ESR is 10 mm in the first hour.

Incorrect - The correct answer is Osteomalacia


Osteomalacia is relatively common in Asian females. The alkaline phosphatase is raised, the
calcium low or normal, with a low serum phosphate. Bone pain can be due to subclinical
fractures.

A 68-year-old white man has been unwell for three months and develops pain over the thoracic
spine. On examination there is evidence of recent weight loss; there is tenderness over T10.
Serum corrected calcium is 3.3 mmol/l, creatinine is 350 mol/l, and ESR is 110 mm in the first
hour.

Incorrect - The correct answer is Multiple myeloma


Multiple myeloma presents commonly in this age group and slightly more commonly in
males. Tenderness over T10 indicates vertebral collapse secondary to lytic bone lesions.
Renal failure may be secondary to hypercalcaemia, hyperuricaemia or dehydration. A
markedly elevated ESR is common due to the presence of paraprotein in the serum.

A 30-year old white man complains of chronic pain in the lumbosacral region. Examination
reveals tenderness over the sacro-iliac joints and restricted range of spinal movements. Serum
corrected calcium is 2.3 mmol/l, alkaline phosphatase is normal, and ESR is 30 mm in the first
hour.

Incorrect - The correct answer is Ankylosing spondylitis


Serum calcium and alkaline phosphatase are normal in ankylosing spondylitis. The ESR is
often elevated. Restricted range of lumbar lateral flexion is often an early feature and
ankylosing spondylitis usually presents below the age of 40.

A 66-year-old man presents with a fracture of the right hip after suffering a fall in the kitchen at
home. In addition to the fracture, the hip x ray shows multiple well-defined lytic lesions in the
pelvic bones and the femur. Urinalysis shows heavy proteinuria.

Incorrect - The correct answer is Multiple myeloma

Again myeloma is the most likely diagnosis in this case. Metastatic prostate carcinoma may
present with lytic lesions and pathological fractures but would not cause proteinuria, which in
this case is likely to indicate Bence Jones protein.

Work Smart
Time taken: 01:41

Theme:Leg weakness
A

Brain stem

Cerebral hemisphere

Conus medullaris

Common peroneal nerve

L5 nerve root

Lumbosacral plexus

Muscle (ie, a myopathy)

Peripheral nerve (ie, a polyneuropathy)

Sciatic nerve

Spinal cord

Sural nerve

Tarsal tunnel

Each of the items below is an extract from the medical history and examination findings of a
patient with leg weakness.
For each one, select the most likely location of the lesion from the list of options.
"... weakness and wasting of the right tibialis anterior and calf, absent right ankle jerk and
extensor plantar responses; nerve conduction studies showed normal motor conduction velocity
and sensory responses from the right common peroneal nerve..."

Incorrect - The correct answer is Conus medullaris


Given normal conduction velocity and sensory responses in the common peroneal nerve,
and the extensor plantar response, with muscle wasting an upper motor neurone lesion is
suggested e.g. compression of the conus medullaris.

" ... right leg weakness of subacute onset involving ankle dorsiflexion, eversion and inversion,
with depression of the ipsilateral ankle jerk and sensory loss confined to the dorsum of the foot
and the anterior aspect of the leg ..."

Incorrect - The correct answer is L5 nerve root


The area of sensory loss gives the site of the lesion, the most likely cause being herniation
of the intervertebral disc between L4 and L5 and compression of the L5 nerve root.

"... weak left leg evolving over a few weeks with tingling in the foot and, on direct questioning,
urinary urgency; examination showed increased tone in both lower limbs and bilaterally extensor
plantar responses; weakness restricted to left hip flexion and ankle dorsiflexion; pain sensation
reduced in the right foot but joint position sense lost in the left foot ..."

Incorrect - The correct answer is Spinal cord


The clinical findings suggest a spastic paraparesis of gradual evolution, the lesion lying in
the spinal cord e.g. multiple sclerosis.

"... gradually evolving difficulty rising from a chair and climbing stairs, without sensory loss;
electromyography of the quadriceps muscle showed small, broken up motor unit potentials in
normal numbers..."

Incorrect - The correct answer is Muscle (ie, a myopathy)


The EMG findings are typical of a myopathy, and the history suggests that it is proximal e.g.
Polymyalgia Rheumatica, osteomalacia, Cushing's disease

"... severe pain in the lower back and left thigh and leg in a patient on anticoagulants; marked
weakness of hip flexion and knee extension on the same side with impaired sensation on the
anterior aspect of the thigh..."

Incorrect - The correct answer is Lumbosacral plexus


This history suggests a haematoma in the lumbar region with pressure effect on the
lumbosacral plexus.

Work Smart
Time taken: 01:48

Theme:Radiological Investigations
A

Chest radiograph

Computed tomography of the brain

Computed tomography of the pelvis

Computed tomography of the thorax

Computed tomography pulmonary angiogram

Echocardiography

Hysterosalpingogram

Intravenous urography

Magnetic resonance imaging of the brain

Radionucleotide ventilation/perfusion scan

Skull radiography

Transcranial ultrasound

Ultrasound of the abdomen

Ultrasound of the pelvis

Each item below is an extract from the clinical information written on a referral card sent to
the radiology department.

For each subject, select from the list of options the imaging investigation most likely to be
informative.
"... 65-year-old male hypertensive; acute central chest pain; a widened superior mediastinum on
his chest radiograph ..."

Incorrect - The correct answer is Computed tomography of the thorax


The presumed diagnosis here must be aortic dissection, and this will be revealed using a CT
of the thorax and aorta. In the past, aortic arch angiography was considered the optimal
diagnostic tool but this has been superseded by imaging with CT or MRI.

"... 24-year-old female non-smoker; pleuritic chest pain and haemoptysis; a normal chest
radiograph ..."

Incorrect - The correct answer is Computed tomography pulmonary angiogram


These clinical features suggest pulmonary embolism and CTPA is regarded as the best test
to diagnose this condition.

"... 24-year-old female; 12 weeks pregnant, with vaginal bleeding ..."

Incorrect - The correct answer is Ultrasound of the pelvis


Ultrasound of the pelvis does not deliver any ionising radiation to the fetus at a critical time in
its development, whilst investigating the cause of the vaginal bleeding, for example,
threatened miscarriage.

"... 22-year-old female, unable to conceive; history of pelvic inflammatory disease ..."

Incorrect - The correct answer is Hysterosalpingogram


The hysterosalpingogram will investigate the anatomy of the uterus and fallopian tubes.
Pelvic inflammatory disease is associated with infertility, due to scarring in the fallopian tubes
secondary to salpingitis.

"... 4-day-old male; convulsions and bulging fontanelles ..."

Incorrect - The correct answer is Transcranial ultrasound


Transcranial ultrasound is an useful tool in the assessment of brain injury in the newborn.

Work Smart
Time taken: 01:55

Theme:CNS PROBLEMS
A

Bacterial meningitis

Cryptococcal meningitis

Encephalitis

Guillian-Barre syndrome

Human immunodeficiency virus (HIV) infection

Listeriosis

Multiple sclerosis

Secondary cancer

Subarachnoid haemorrhage

Viral meningitis

All the patients described below have had a lumbar puncture.


For each one, choose the single most likely diagnosis from the list of options.
Each option may be used once, more than once, or not at all

A 32-year-old doctor with a family history of polycystic disease of the kidney collapsed suddenly
after a sudden persistent occipital headache. A sample of cerebrospinal fluid obtained 12 hours
later was reported as xanthochromic.

Incorrect - The correct answer is Subarachnoid haemorrhage

A 28-year-old woman presents with urinary incontinence and pain on movement of right eye with
rapid deterioration in central vision. On examination she has impaired co-ordination on heel-shin
test. She has nystagmus and an internuclear ophthalmoplegia. The cerebrospinal fluid shows a
slight increase in lymphocyte count, raised total proteins and raised immunoglobulins.

Incorrect - The correct answer is Multiple sclerosis

An 18-year-old student presents with headache, neck stiffness and photophobia. The
cerebrospinal fluid examination shows 100 lymphocytes, CSF glucose is more than 2/3 blood
glucose value and CSF protein is 0.60g/L. Gram stain was negative.

Incorrect - The correct answer is Viral meningitis

A 56-year-old woman has a history of headaches for several weeks. More recently she has had
several convulsions. She was a heavy smoker until six years ago. She has recently experienced
moderate weight loss. Cerebrospinal fluid shows increased lymphocytes, with clumps of irregular
cells which have deeply hyperchromatic nuclei and scanty cytoplasm.

Incorrect - The correct answer is Secondary cancer

A 24-year-old student has a 24 hour history of an ear infection, with photophobia, neck stiffness
and a headache. Cerebrospinal fluid shows a white cell count of 500/mm 3, almost all of which are
polymorphs.

Correct

The 32-year-old patient has subarachnoid haemorrhage. The incidence of subarachnoid


haemorrhage is 15/10000. The age range is typically 35-65 years. Common causes are
rupture of congenital berry aneurysms in 70% of patients, and arterovenous malformations
in 15% of patients.
Recognised associations include
Polycystic kidney disease
Ehlers-Danlos syndrome and
Coarctation of the aorta.

Sudden severe occipital headache is the most striking clinical feature and may be
associated with focal neurological signs. CT brain scan is indicated and if no bleed is
identified a lumbar puncture may identify blood stained fluid or xanthochromia.
The 28-year-old patient has multiple sclerosis. Multiple sclerosis is a chronic relapsing and
remitting disorder characterised by demyelinating plaques within the CNS. Clinical features
may be wide ranging including CNS defects such as
Spastic paraparesis
Cerebellar signs
Optic atrophy
Nystagmus
Internuclear ophthalmoplegia and
Urinary incontinence.

CSF examination shows


Raised protein and lymphocyte count
Oligoclonal bands of IgG on CSF electrophoresis and
Delayed visual, auditory and somatosensory evoked potentials.

MRI is sensitive but not specific for plaque detection.

The 18-year-old patient has viral meningitis. Causative viruses include


Herpes simplex
Varicella zoster
Coxsackie
Echo
Mumps and
Influenza viruses.

the 56-year-old patient has secondary cancer. Secondary carcinomas form approximately
25% of all CNS malignancies. These commonly originate from the bronchi, breasts,
stomach, prostate, thyroid or kidney.
The 24-year-old patient has bacterial meningitis. Bacterial meningitis usually has a rapid
onset of less than 48 hours. Meningococcus, pneumococcus and Haemophilus are the
common causes of pyogenic infection.

Work Smart
Time taken: 02:03

Theme:Causes of Breathlessness
A

Acute blood loss

Asthma

Atypical pneumonia

Bronchiectasis

Bronchopneumonia

Carcinoma of the bronchus

Centrilobular emphysema

Extrinsic allergic alveolitis

Mesothelioma

Chronic anaemia

Sarcoidosis

Each patient described below presents with breathlessness. For each one, choose the single
most likely diagnosis from the list of options.
Each option may be used once, more than once, or not at all.
A 60-year-old builder has smoked 30 cigarettes each day for 40 years. He has recently
developed breathlessness associated with cough and haemoptysis. There is clubbing of the
fingers and toes. On percussion, there is stony dullness over the left side of the chest.

Incorrect - The correct answer is Carcinoma of the bronchus

Cigarette smoking is the major risk of bronchial cancer. The risk is dependent on the number
of cigarettes and duration of smoking. 80% of patients present with chronic cough, 70% with
haemoptysis, and 15% with recurrent or slowly resolving pneumonia. Intrathoracic
complications include pleural effusions, recurrent laryngeal nerve palsies, SVC obstruction
and Horner's syndrome.

A 64-year-old former shipyard worker, a lifelong non-smoker, complains of increasing difficulty


with breathing. There is a persistent dull ache in the left chest, which is exacerbated on deep
inspiration. On examination, there is reduction in respiratory movement on the left; a chest x-ray
shows lobular pleural thickening.

Incorrect - The correct answer is Mesothelioma

Mesothelioma is a pleural based lung malignancy which is related to earlier asbestos


exposure usually from workplace, such as ship yards. Clinical features include chest pain,

dypsnoea and blood stained pleural effusion. Diagnosis is by pleural biopsy. Prognosis is
poor and treatment is symptomatic.

A 42-year-old crop farmer complains of persistent breathlessness that has developed over
several years. He has exacerbations of breathlessness when he handles hay, associated with
fever and malaise.

Incorrect - The correct answer is Extrinsic allergic alveolitis

Farmer's lung is due to a hypersensitivity reaction to Micropolyspora faeni. Clinical features


occur 4-8 hours after exposure; fever, malaise, dry cough and dypsnoea are usual. Chronic
disease may follow acute symptoms or occur independently. In the acute stage, neutrophilia,
high ESR, and positive serum preciptins are usual. Chronic disease shows upper zone
shadowing due to fibrosis. Prednisolone is the treatment of choice.

An 18-year-old cystic fibrosis sufferer has persistent cough, which is productive of purulent
sputum. He has finger clubbing and low-pitched inspiratory and expiratory crackles on
auscultation.

Incorrect - The correct answer is Bronchiectasis

Cystic fibrosis is a cause of a bronchiectasis. Clinical features of bronchiectasis include


persistent productive cough (especially in winter months), haemoptysis, clubbing and lowpitched inspiratory and expiratory crackles on auscultation. CXR shows cystic shadows, fluid
levels, and tramline or ring shadows.

A 48-year-old woman of Caribbean origin complains of progressive shortness of breath and


painful lesions on her shins. She has a history of hypertension and of joint pain. A blood test
shows hypercalcaemia.

Incorrect - The correct answer is Sarcoidosis

Sarcoidosis is of unknown cause characterised by non-caseating granuloma. It is commoner


in Afro-Caribbean people and it may affect any organ or age group.

Work Smart
Time taken: 02:11

Theme:Investigation of emergencies
A

Arterial blood gases

Blood glucose

Blood urea electrolytes and creatinine

CT scan of brain

Full blood count and group and cross match

MRI scan of knees

Thyroid function tests

Toxicology screen

Skull x ray

For each patient below, choose the single most essential diagnostic investigation from the
given list of options.
Each option may be used once, more than once or not at all.
A 21-year-old female is found unconscious next to her 22-year-old husband, who was found
dead. Her electrocardiogram (ECG) shows evidence of acute myocardial infarction (MI).

Incorrect - The correct answer is Toxicology screen


Severe cases of cocaine intoxication may result in:
Acute myocardial infarction
Aortic dissection
Myocarditis
Ventricular arrhythmias

Cardiorespiratory arrest.

A 24-year-old female, admitted to the Emergency department with pneumonia is now barely
conscious, with poor respiration, despite high flow oxygen therapy.

Correct
This patient needs urgent arterial blood gas estimation, and should be considered for
artificial ventilation.

A 21-year-old rugby player, who had suffered from an episode of brief unconsciousness. He is
rushed into the Emergency department, unconscious, the next day.

Incorrect - The correct answer is CT scan of brain


Extradural haemorrhage is suggested as complicating a head injury; a period of
unconsciousness is followed by a period of alertness and the rapid deterioration into
unconsciousness.

A 72-year-old woman is admitted unconscious with a core temperature of 35.6C. She has a
heart rate of 42 beats per minute and slowly relaxing reflexes.

Incorrect - The correct answer is Thyroid function tests


Drowsiness, bradycardia and slowly relaxing reflexes would suggest the diagnosis of
hypothyroid coma. Urgent thyroid function tests will confirm the diagnosis.

A 24-year-old school teacher, who is a very well controlled diabetic, is found unconscious by her
students after lunch. On her desk is a pile of partly marked papers and an uneaten sandwich.

Incorrect - The correct answer is Blood glucose


Urgent blood glucose estimation by sampling capillary blood would confirm the diagnosis of
hypoglycaemic coma. However, she requires urgent administration of 50% dextrose or a
glucagon injection.

Work Smart
Time taken: 02:23

Theme:Acute poisoning
A

Benzodiazepines

Digoxin

Ecstasy

Opioids

Organophosphorus compounds

Paraquat

Paracetamol

Salicylates.

Warfarin

For each of the patients below choose the single most likely cause of their presentation from
the given options.
Each option may be used once, more than once or not at all.
A 60-year-old farmer is rushed into the Emergency department unconscious, sweating profusely
and with copious secretions drooling from his mouth. His pupils are small.

Incorrect - The correct answer is Organophosphorus compounds


Organophosphorus compounds used as insecticides or military 'nerve gases' act by
inhibiting acetylcholinesterase. Clinical features include:
Miosis
Blurred vision
Hypersalivation
Expectoration of frothy secretions

Nausea
Vomiting
Abdominal cramps
Diarrhoea
Bronchospasm.

Diagnosis can be confirmed by measuring the plasma or RBC cholinesterase activity.


Treatment is supportive. Specific antidotes to organophosphate poisoning are atropine and
cholinesterase reactivators (for example, oximes such as pralidoxime).

A 28-year-old is brought to the Emergency department on Friday night. Examination reveals


hyperthermia, hyperglycaemia and hypertension.

Incorrect - The correct answer is Ecstasy


3,4- methylenedioxymethamphethamine (MDMA), commonly known as 'ecstasy'. Clinical
features in most cases of mild abuse are characterised by:
Agitation
Tachycardia
Hypertension
Widely dilated pupils
Trismus
Sweating.

In more severe cases, hyperthermia, disseminated intravascular coagulation,


rhabdomyolysis and acute renal failure predominate. Treatment in severe cases involves the
use of intravenous fluids and, if necessary, dantrolene, 1 mg/kg body weight
intravenously,should be administered and repeated as necessary to reduce hyperthermia.

A 74-year-old woman is brought into the Emergency department with melaena and bruising. She
is semiconscious.

Incorrect - The correct answer is Warfarin


Management of severe warfarin toxicity involves use of whole blood, fresh frozen plasma or
clotting factor concentrates may be required in severe acute haemorrhage, but vitamin K
given until clotting returns to normal is usually sufficient.

An 18-year-old male, brought with history of consumption of unknown substance. Investigation


reveals he is unconscious, barely breathing, normal pulse and small pupils.

Incorrect - The correct answer is Opioids


Opiod toxicity may manifest as coma, very low respiratory rate and pin-point pupils. Other
complications include:
Convulsions
Hypotension
Peripheral circulatory failure
Cardiac arrhythmias and conduction defects
Hypothermia
Pulmonary oedema
Renal failure
Rhabdomyolysis.

Resuscitation comprises establishing a clear airway, giving oxygen, placing the patient in a
semi-prone position to reduce the risk of aspiration in the event of vomiting, and immediate
intravenous injection of an adequate dose of naloxone. Assisted ventilation may be required
if naloxone is not immediately available or if very large doses are required.

An 80-year-old man who has a history of heart disease has nausea, vomiting, hypokalaemia and
cardiac arrhythmias.

Incorrect - The correct answer is Digoxin


Binding of digoxin to the Na+/K+-ATPase transport system is inhibited by high levels of
potassium and the level of activity of this enzyme is increased by the presence of
magnesium. Thus both hypokalaemia and hypomagnesaemia increase digoxin toxicity.
Digoxin toxicity is more common in patients than other drugs such as beta blockers,
verapamil and quinidine. Hypothyroid patients are more sensitive to the effects of digoxin.

Cardiac monitoring should be undertaken. Normal saline is the intravenous fluid of choice.
Hypokalaemia and hypomagnesaemia should be corrected.
Administration of digoxin Fab is the primary treatment for all the major cardiac complications
of heart block, arrhythmias, hyperkalaemia of more than 6 mmol/l or digoxin level of more
than 10 nmol/L (six hours after ingestion). If this is unavailable, heart block should be treated
with pacing and tachyarrhythmias may be treated with magnesium.

Work Smart
Time taken: 02:27

A 4-month-old boy is brought in dead to hospital.


He had had a cold for three days, with crusty nose and mild fever. He went to bed at 7 pm as
usual. Mother checked him at 11 pm before going to bed. In the morning she found him stiff
and cold.
He was brought to the Emergency department by ambulance, but resuscitation was
unsuccessful. Mother is single 19 years and smokes 20 per day.
He was born at 39/40 weighing 3.25 kg, and there were no neonatal problems. He had been
growing along the 50th centile for height and weight.
Which of the following is not a risk factor for sudden infant death syndrome?
(Please select 1 option)
Intercurrent infection
Maternal smoking
Over-wrapping
Prematurity
Supine sleeping position

Correct

This is a typical history of SIDS.


The official definition is: "The sudden death of an infant under one year of age that remains
unexplained after a thorough case investigation, including performance of a complete
autopsy, examination of the death scene, and a review of the clinical history."
Risk factors include:
maternal smoking

prematurity
over-wrapping
intercurrent infection
prone sleeping position
low social class.

Work Smart
Time taken: 02:31

Which of the following is an indicator of poor prognosis in acute pancreatitis?


(Please select 1 option)
Albumin = 35 g/ L

ALT = 130 IU/L

Calcium = 1.91 mmol/L

This is the correct answer

Creatinine = 130 mmol/L

Pa O2 = 10 kPa

Incorrect answer selected

Poor prognostic indicators in the first 48 hours of acute pancreatitis include:


Age more than 55 years
White cell count (WCC) more than 15
Glucose more than 10
Urea more than 16
Albumin less than 30 g/L
Alanine aminotransferase (ALT) more than 200

Calcium less than 2


Lactate dehydrogenase (LDH) more than 600 and
Partial pressure of oxygen in arterial blood (PaO2) less than 8.

Work Smart
Time taken: 02:36

An 80-year-old retired haulier with a history of controlled hypertension presents with acute
onset weakness of his left arm, that resolved over 12 hours. He had suffered two similar
episodes over the last three months.
Examination reveals a blood pressure of 132/82 mmHg and he is in atrial fibrillation with a
ventricular rate of 85 per minute. CT brain scan is normal.
What is the most appropriate management?
(Please select 1 option)
Amiodarone
Aspirin
Digoxin
Dipyridamole
Warfarin

Correct

It seems that this patient has had three transient ischaemic attacks due to atrial fibrillation.
The most appropriate therapeutic strategy would be warfarin.
Studies reveal that warfarin would be therapeutically superior to aspirin in such a patient's
case, provided he is not at risk of falls, when the risk of a traumatic bleed may outweigh the
benefits of anticoagulation.

Work Smart
Time taken: 02:42

Which of the following drugs is matched correctly with its action?


(Please select 1 option)
Amiodarone shortens the action potential

Incorrect answer selected

Digoxin stimulates Na/K ATPase


Simvastatin inhibits HMGCoA reductase

This is the correct answer

Sotalol has class I and class III properties


Streptokinase increases fibrinogen levels

Sotalol has class II (beta-blocker) and class III (amiodarone) like actions.
Amiodarone prolongs the action potential and the QT interval.
Both sotalol and amiodarone carry the risk of precipitating ventricular arrhythmias.
After thrombolysis, fibrinogen levels are low, due to consumption during thrombosis.
Digoxin inhibits the cardiomyocyte membrane Na/K ATPase

Work Smart
Time taken: 02:49

An 18-year-old college student is admitted with a two week history of paraesthesia and
weakness affecting all four limbs.
Examination shows mild distal weakness, absent reflexes, and glove and stocking sensory
loss. FVC is 900 mls. CT head is normal. Lumbar puncture reveals 5 lymphocytes, 1 red cell,
glucose 4.5 and protein of 0.9 g/l.
What is the next management step?
(Please select 1 option)
Intravenous immunoglobulin
Intubation and ventilation
MRI brain
Plasma exchange

This is the correct answer

Serum lead levels

Incorrect answer selected

The clinical features and CSF findings are typical of Guillain-Barre syndrome.
An forced vital capacity (FVC) below 1 litre (or about 15 mls per kilogram body weight) is
indicative of severe respiratory weakness and is an indication for urgent intubation and
ventilation.
The specific treatment is either intravenous immunoglobulin or plasma exchange.

Work Smart
Time taken: 02:53

In the treatment of an individual with type 1 diabetes who presents in DKA (diabetic
ketoacidosis) with a pH of 7.1, which of the following statements is correct?
(Please select 1 option)
Bicarbonate should be given immediately
DKA in pregnancy carries a significant mortality to the to the fetus

This is the correct answer

Hyperphosphataemia occurs following treatment


Serum osmolality will not help in monitoring response to treatment
The glucose level correlates well with the degree of acidosis

Incorrect answer selected

Diabetic ketoacidosis in pregnancy carries an increased mortality to both mother and fetus.
Serum osmolality is more reliable than glucose.
Phosphate and potassium fall with treatment.
The use of bicarbonate is not evidence based in any robust way, there is no threshold at
which there is evidence for its use, and if given at all, it should be given with care, with close
monitoring of the patient

Work Smart
Time taken: 03:00

A 64-year-old teacher reaches casualty resuscitation department after experiencing 30


minutes of severe central chest pain radiating to the left arm.
The initial ECG shows 1 mm ST elevation in leads V1 - V3 only.
He is given oxygen and morphine analgesia.
Which of the following is the most appropriate next step?
(Please select 1 option)
Emergency referral to cardiologist for primary angioplasty
Aspirin, clopidogrel and repeat ECG in 15 minutes

This is the correct answer

Transfer to coronary care unit immediately


No further therapy until 12 hour troponin
Aspirin, clopidogrel and thrombolysis

Incorrect answer selected

The symptoms are suspicious for acute myocardial infarction, however, he does not fulfil
criteria for anterior ST elevation MI (2 mm ST elevation in contiguous chest leads or new left
bundle branch block [LBBB]) so primary angioplasty/thrombolysis is not indicated at this
stage.
He should be given aspirin and clopidogrel and the ECG repeated in 15 minutes for evolving
changes. If subsequent ECG meets criteria and pain persists, he should be referred for
primary angioplasty.
If the second ECG does not meet criteria and he is pain free, he should be treated as a nonST elevation acute coronary syndrome, with clexane, clopidogrel etc. and closely monitored
on coronary care unit.

Work Smart
Time taken: 03:04

A 61-year-old West Indian presents to casualty with sudden onset of leg weakness
associated with back pain.
On examination there is flaccid paraplegia with absent reflexes and silent plantars, and a
sensory level at T10 with preservation of dorsal column modalities.
MRI of the spine performed the morning after admission is normal.
Which of the following is the most appropriate next investigation?

(Please select 1 option)


CSF analysis
CT chest with contrast

This is the correct answer

CT head
Serum B12 estimation
Spirometry

Incorrect answer selected

The clinical picture here is that of acute anterior cord syndrome (dorsal columns preserved),
and is often caused by cord infarction.
Occasionally, especially in hypertensive Afro-Carribean patients, and in the presence of
chest or back pain, this is caused by thoracic aortic dissection interrupting the anterior spinal
artery.
This possibility needs to be pursued with a chest CT scan.

Work Smart
Time taken: 03:07

Which of the following is not a recognised cause of a dominant R in lead V1 on the ECG?
(Please select 1 option)
Dextrocardia

Duchenne muscular dystrophy

Ebstein's anomaly

Myotonic dystrophy

This is the correct answer

Primary pulmonary hypertension

Incorrect answer selected

A dominant R wave in V1 has few causes


Right ventricular hypertrophy (RVH) as in Eisenmenger's syndrome

Primary pulmonary hypertension (HT)


Right bundle branch block (RBBB) as in Ebstein's anomaly
Wolff-Parkinson-White syndrome type A
Dextrocardia
True posterior myocardial infarction (MI)
Duchenne muscular dystrophy.

Work Smart
Time taken: 03:11

A 16-year-old A-level student develops deterioration in vision in her left eye over three days.
She complains of discomfort in the eye and thinks that difficulty with perception of colour was
the first problem that she noticed, during a trip to an art gallery.
On examination, visual acuity on the left is down to light perception. The pupil appears
dilated and does not constrict to light, although does when a torch is shone in the right eye.
What is the most likely diagnosis?
(Please select 1 option)
Amaurosis fugax

Anterior ischaemic optic neuropathy

Central retinal artery occlusion

CMV retinitis

Optic neuritis

Correct

The causes of acute unilateral loss of vision include:

Open angle glaucoma - other eye is often affected


Giant cell arteritis - associated with headache, jaw claudication, scalp tenderness. High
ESR. Usually affects both eyes
Retinal artery occlusion - acute, with maximal deficit at the onset and variable resolution,
and a spot at the macula
Amaurosis fugax - transient blindness lasting minutes to hours. Associated with vascular
risk factors as usually embolic. May be described as a "curtain coming down"
Anterior optic neuropathy - painless, may be central field defect with colour disturbance a
prominent feature. Arteriosclerosis, hypertension and diabetes are risk factors
Optic neuritis - painful, colour vision affected. May not result in complete visual loss. May
be the first manifestation of MS, therefore requires appropriate investigation
Other causes - cytomegalovirus (CMV), toxoplasmosis in association with
immunodeficiency.

Work Smart
Time taken: 03:12

A 16-year-old female presents with shortness of breath. She is known to suffer from asthma
and her usual best PEFR is 410 L/min.
Which of the following features would suggest that this is a severe asthma attack?
(Please select 1 option)
Blood pressure of 156/80 mmHg

PEFR 200 L/min

This is the correct answer

Pulse of 100 beats per minute

Respiratory rate of 22 per minute

Temperature of 39C

Incorrect answer selected

Guidelines from the British Thoracic Society suggest that the following are suggestive of
acute severe asthma:
inability to complete sentences
a tachycardia in excess of 110 bpm
a respiratory rate above 25/minute, and
a PEFR 33-50% of the predicted value.

Life-threatening asthma would include:


silent chest
bradycardia
hypotension, and
hypoxia.

Work Smart
Time taken: 03:11

A 7-year-old girl presents with high fever and severe left-sided throat pain.
She has had difficulty in swallowing over the last two days, and has been finding it
increasingly uncomfortable to open her mouth. Her voice is muffled and she
dribbles saliva.
She was born at 41/40 gestation weighing 4.0 kg and there were no neonatal
problems.
On examination she looks ill. Her temperature is 40.2C (tympanic), RR 15/min and
HR 100/min. ENT examination shows left tonsillar enlargement and exudate, with a
uvula deviated to the right.
What is the most likely diagnosis?
(Please select 1 option)
Atypical tuberculosis

Cervical lymphangitis
Foreign body aspiration
Peritonsillar abscess

This is the correct answer

Retropharyngeal abscess

Incorrect answer selected

The history suggests a peritonsillar abscess (quinsy) on the left side, a complication
of Group A streptococcus sore throat.
A fever greater than 39.4C is associated with severe disease, and treatment is by
surgical drainage.

Work Smart
Time taken: 03:15

A 14-year-old girl was found unconscious at the roadside by a passer-by, who called an
ambulance.
On examination she was unkempt, had slurred speech and an ataxic gait. Her core
temperature was 34.5C, but there were no other specific abnormalities to find. Her BM stix
result was 2.9 mmol/L.
What is the most likely diagnosis?
(Please select 1 option)
Alcohol ingestion

This is the correct answer

Complex partial seizure


Encephalitis
Insulin overdose
Migraine

Incorrect answer selected

The picture suggests acute alcohol ingestion. This is often accompanied by hypothermia and
hypoglycaemia.
It is important to exclude other ingestions and to look for coexisting social problems.

Work Smart

Time taken: 03:19

A 3-year-old child presents with bruising to the emergency department. His mother says that
this appeared yesterday.
He was born at 32+2 weeks gestation weighing 1.54 kg. He required ventilation for
surfactant deficient lung disease, then home oxygen for chronic lung disease. He was
operated on for necrotising enterocolitis.
Subsequently he was admitted with failure to thrive and his mother was treated for postnatal
depression. He is one of three children, all under five years. Both parents are unemployed.
On examination he looks weepy and suspicious. He is on the second centile for height and
the 0.4th centile for weight. He has Harrison's sulcus. Temperature is 36.5C, respiratory
rate 20/min and heart rate 95/min. Chest is clear and there is no murmur. He has three
purple linear bruises over his back and thighs 5, 5.5 and 7 cm long.
What is the most likely diagnosis?
(Please select 1 option)
Acute lymphoblastic leukaemia
Child physical abuse

This is the correct answer

Henoch-Schnlein purpura
Idiopathic thrombocytopenic purpura
Meningococcal septicaemia

Incorrect answer selected

This is a vulnerable child with fresh bruising and an inconsistent history. The pattern of
bruising suggests the use of a strap or belt. The diagnosis is child physical abuse until
proven otherwise.
It is important to familiarise yourself with the local procedures for managing such cases.
Involve your seniors. Remember to record and sign your findings in detail, including verbatim
record of what the mother has said, measurements and photographs of physical findings.
Clotting should be checked and consideration given to doing a skeletal survey.

Work Smart
Time taken: 03:29

An 8 month old child presents with spots on the legs. He is well and feeding well. 39+6/40
3.5 kg, no neonatal problems. No drugs nor medications, fully immunised. No FH/SH of note.
On examination temperature 37.6C (tympanic), RR 30/min, HR 110/min. Well perfused,
capillary refill time of 1 second. There are 20-30 1-2 mm non-blanching purpuric spots over
the shins.
What is the most likely diagnosis?
(Please select 1 option)
Child physical abuse
Cough petechiae
Enteroviral infection

This is the correct answer

Henoch-Schnlein purpura
Idiopathic Thrombocytopaenic purpura

Incorrect answer selected

This child is well, and presents with purpuric spots and a low-grade fever. Although about
20% of such children have serious bacterial infection and 7-10% have meningitis/
septicaemia, this still leaves 70% who have some sort of viral infection.
A large number of viruses (eg Varicella and EBV) can present in this way, although in clinical
practice the specific cause is rarely found

Work Smart
Time taken: 03:33

A 14-day-old boy is admitted with collapse.


He was born at 38+2/40 weighing 3.47 kg to a healthy Caucasian mother. Pregnancy and
delivery had been uncomplicated.
Following discharge he had failed to gain weight, and began vomiting three days ago. This
had persisted and worsened. Over the past two days the nappies had been wet twice a day.
On examination temperature 36.2C (tympanic), heart rate 160/min (thready pulses),
Respiratory rate 40/min. No murmur, chest clinically clear. Abdomen soft with no enlarged
liver, spleen or kidneys. Oxygen saturations are 85% in air.
Blood tests show:

Sodium

127 mmol/L

(137-144)

Potassium

5.8 mmol/L

(3.5-4.9)

Urea

13.7 mmol/L

(2.5-7.5)

Creatinine

74 mol/L

(60-110)

What is the most likely diagnosis?


(Please select 1 option)
Congenital adrenal hyperplasia

This is the correct answer

Critical aortic stenosis


Galactosaemia
Group B streptococcal infection
Hypoplastic left heart syndrome

Incorrect answer selected

The picture is one of shock in a child who has become progressively dehydrated.
Sepsis and congenital heart disease are possible, but there is nothing specific to point in
these directions.
The electrolytes make the likely diagnosis salt-losing congenital adrenal hyperplasia.

Work Smart
Time taken: 03:36

An 18-month-old girl presents with stridor at 1 am. She has had a cold for 48 hours, with lowgrade fever, but went to bed as usual at 7.30 pm. She awoke four hours later crying and
distressed, with a barking cough.
What is the most likely diagnosis?
(Please select 1 option)
Aspiration of foreign body
Asthma
Bacterial tracheitis
Croup

This is the correct answer

Epiglottitis

Incorrect answer selected

This is a classical history of viral croup.


Most of these episodes are one-off and settle rapidly without treatment or with
dexamethasone orally

Work Smart
Time taken: 03:41

A 13-year-old girl is rushed into hospital, having become rapidly drowsy after running the
1500 metres in the school athletics competition.
She has been on insulin for diabetes for three years. Her latest HbA1c is 62 mmol/mol.
On examination she responds to pain, and is pale and sweaty. Her temperature is 36.5C,
Respiratory rate 12/min and heart rate 80/min. There are no focal neurological findings.
What is the most likely diagnosis?
(Please select 1 option)
Hypoglycaemia

This is the correct answer

Postural hypotension
Substance abuse
Vasovagal episode
Viral encephalitis

Incorrect answer selected

The history suggests tight diabetic control, with neurological deterioration following exercise.
Hypoglycaemic coma is most likely, and the sweatiness and pallor are suggestive.
The BM stix in this case was 1.2 mmol/L and she recovered rapidly with a bolus of IV 10%
dextrose.

Work Smart
Time taken: 03:47

A 7-year-old girl with cystic fibrosis Is brought by her mother to the Emergency department
following a massive haematemesis.
Examination reveals that she is hypotensive and she has a tachycardia.
What is the most likely diagnosis?
(Please select 1 option)
Aorto-intestinal fistula
Bleeding oesophageal varices

This is the correct answer

Boerhaaves syndrome
Mallory-Weiss tear
Perforated peptic ulcer

Incorrect answer selected

The most likely cause for this type of large upper gastrointestinal haemorrhage causing
shock in a child of 7 years would be from bleeding oesophageal varices secondary to portal
hypertension.
Perforated ulcer is less likely.
Mallory Weiss tear would not cause such catastrophic bleeding.
Aorto-intestinal fistula is seen in older patients with aortic abdominal aneurysm.
Boerhaave's syndrome is due to oesophageal rupture normally after severe vomiting.

Work Smart
Time taken: 03:52

Theme:Childhood Hypertension
A

DMSA scan

Echocardiogram

Plasma renin and aldosterone

Renal angiography

Thyroid function tests

Urine catecholamines

Urine drug screen

U+Es, creatinine

Ultrasound KUB

For each of the following scenarios choose the most specific investigation:
A 15-year-old girl is brought to the Emergency department having collapsed at a party. On
examination she has a dry mouth, a heart rate of 110/min and blood pressure of 150/110 mmHg.

Incorrect - The correct answer is Urine drug screen

In the 15-year-old girl there is acute hypertension, most likely related to drugs. The most
common offender these days is probably MDMA (ecstasy). A drug screen will confirm this.

A 6-year-old boy presents with headache on defecation. On examination his blood pressure in
the right arm is 140/100 mmHg and he has a 2/6 systolic murmur.

Incorrect - The correct answer is Echocardiogram

The 6-year-old boy has hypertension and a heart murmur. It is important to exclude
coarctation. The murmur for this may be loudest over the scapula. Four limb BPs are usually
diagnostic, but echocardiography will identify the precise structural lesion.

A 2-month-old infant is found to have a blood pressure of 120/90 mmHg persistently. He was
born at 25/40 gestation, was ventilated for 14 days and is still on oxygen.

Incorrect - The correct answer is Renal angiography

The 2-month-old infant is likely to have renal artery thrombosis complicating umbilical artery
catheter (UAC) insertion. Renal angiography will confirm this.

In evaluating raised blood pressure:


First decide whether it is acute or chronic
Second whether it is related to drugs
Third is it localised?

Work Smart
Time taken: 04:03

Theme:Investigation of wheeze
A

Allergy testing

Barium swallow

Ciliary motility

CT scan chest

Exercise test

Fluoroscopy

Methacholine challenge

pH Studies

Sweat test

For each scenario choose the most specific diagnostic test.

A 3-week-old infant presents with a history of increasing breathlessness since birth. There is
decreased air entry over the right base.

Incorrect - The correct answer is CT scan chest


This is a child with progressive breathlessness and focal decreased air entry. This is likely to
be due to a mass lesion, such as cystic adenomatous malformation. The chest x ray may
confirm this, but a CT scan of chest will be definitive.

A 3-month-old boy presents with recurrent chestiness and wheeze. He was born at term
weighing 3.7 kg and is now 4.3 kg.

Incorrect - The correct answer is Sweat test


This boy has associated failure to thrive, so cystic fibrosis should be excluded. In older
children with sinus/ear infections -/+ situs inversus, ciliary motility disorders should be
considered.

A 15-month-old boy presents with sudden onset of coughing, lacrimation and wheeze. He has
previously been completely well.

Incorrect - The correct answer is Fluoroscopy


This child's presentation is associated with choking suggestive of foreign body aspiration.
Fluoroscopy or microlaryngobronchoscopy are most helpful here, as many inhaled objects
are not radio-opaque and fail to show up on standard chest x ray.

Wheeze may be acute or chronic. Acute wheeze may be associated with infection or foreign
body. Chronic wheeze may be associated with a normal or abnormal chest x ray.

Work Smart
Time taken: 04:08

Theme:Breathing
A

Intubation

IV coamoxiclav

IV penicillin

Mask ventilation

Oral erythromycin

Oropharyngeal airway

Oral penicillin V

Oxygen via facemask

Oxygen via headbox

For each scenario choose the most appropriate mode of immediate management:
A 3-month-old boy presents with a 3 day history of coryza, cough and progressive difficulty in
feeding. Respiratory rate is 60/min with moderate recession, wheeze and crackles. Heart rate is
150/min, good peripheral perfusion. Oxygen saturation is 88% on air.

Incorrect - The correct answer is Oxygen via headbox


This child has bronchiolitis, is hypoxic and requires oxygen. This is best provided by
headbox or nasal prongs.

A 6-week-old boy presents with bilateral purulent conjunctivitis and difficulty in breathing.
Respiratory rate is 50/min, mild recession. Heart rate is 120/min. Oxygen saturation is 94% on
air.

Incorrect - The correct answer is Oral erythromycin


This boy has chlamydia pneumonitis and conjunctivitis, is not hypoxic, and requires
erythromycin.

A 3-year-old boy presents with fever and breathing difficulty. On examination he has a respiratory
rate of 55/min, painful inspiration, and crackles at the right base. Oxygen saturations are 88% on
air and temperature is 39.9C.

Incorrect - The correct answer is IV penicillin


This boy has bacterial pneumonia with focal consolidation, most likely pneumococcal. IV
penicillin is the treatment of choice.

Work Smart
Time taken: 04:13

A 70-year-old woman, with metastatic breast cancer presented with a week's history of
severe weakness, obtundation, and poor oral intake.
One month ago, CT of the head revealed multiple intracerebral lesions. She underwent
cranial irradiation and received dexamethasone, 12 mg orally daily.
On examination she is unwell and disoriented. Temperature is 36.6C, pulse is 100/min,
respiratory rate is 28/min and blood pressure is 110/60 mmHg supine. Chest examination
and heart examination are normal. There is lower abdominal tenderness, especially in the
suprapubic area. Diffuse muscle weakness is noted. No lateralising neurologic signs or
abnormal reflexes are noted.
Investigations reveal:
White cell count

19.5 109/L

(4-11)

Plasma glucose

40 mmol/L

(3.0-6.0)

Urea

25 mmol/L

(2.5-7.5)

Creatinine

160 mol/L

(60-110)

Calcium

2.2 mmol/L

(2.2-2.6)

Sodium

130 mmol/L

(137-144)

Potassium

5.0 mmol/L

(3.5-4.9)

Bicarbonate

24 mmol/L

(20-28)

Urinalysis

Glucose +++
Protein ++

Moderate bacteria seen

Cultures of blood and urine are requested and she is treated with an intravenous sliding
scale insulin.
Which of the following IV fluids would you prescribe in conjunction with the insulin sliding
scale for this patient?
(Please select 1 option)
5% Dextrose
Dextrose saline
Haemacel
Half strength normal saline
Normal saline

Correct

This patient has an excessive hyperglycaemia contributing to her symptoms and related to
hyperosmolarity, although she is dehydrated rather than has hyperosmolar non-ketotic coma
(HONK).
Also her normal bicarbonate argues against diabetic ketoacidosis.
She needs IV normal saline together with insulin for her crystalloid requirements switching to
IV dextrose when the blood glucose descends to 10 mmol/L.
Her dexamathasone has contributed to this marked hyperglycaemia causing marked insulin
resistance.

Work Smart
Time taken: 04:16
PLAB 2005 Sept

A 22-year-old golf course groundsman develops sudden onset of breathlessness and rightsided chest pain whilst maintaining the 18th hole. He is rushed to hospital, but his condition
worsens when being examined by a junior doctor.
She reported finding a trachea that was deviated to the left, and hearing very distant breath
sounds over his right lung. Following the intervention of a senior doctor, his condition
improves rapidly.
What is the most probable diagnosis?

(Please select 1 option)


Myocardial infarction
Pericarditis
Pneumonia
Pulmonary embolism
Tension pneumothorax

Correct

Sudden onset of chest pain and respiratory distress in a previously fit young man should
alert one to the diagnosis of pneumothorax.
Tales of patients who developed a clinically obvious tension pneumothorax in hospital are
told frequently, so it is best to be able to recognise one if it occurs while you are actually
examining the patient!

Work Smart
Time taken: 04:20
PLAB 2005 Sept

An 18-year-old male is rushed into the Emergency department by a group of friends who
leave before they can be interviewed by medical staff.
He is semiconscious, has a respiratory rate of 8/min, a blood pressure of 120/70 mmHg and
a pulse of 60 bpm. He is noted to have needle track marks on his arms, and his pupils are
small.
What single initial treatment would you administer?
(Please select 1 option)
Adrenaline
Aspirin
Atropine
Dextrose
Naloxone

Correct

These features of unconsciousness, respiratory depression plus small pupils suggest opiate
toxicity. This commonly occurs as the result of the abuse of street drugs such as heroin.
It should be relieved promptly with naloxone which is an opiate receptor antagonist.

Naloxone has a shorter half life than many opiates, so more than one dose may be
necessary.

Work Smart
Time taken: 04:24

A 15-year-old girl attends with her mother as an emergency to the Emergency department.
Her mother provides a history of 30 minutes of deteriorating breathlessness and facial
puffiness. She has otherwise been well except for eczema and she takes the oral
contraceptive. She is in obvious distress, her breathing is laboured and noisy.
What is the single most likely cause of breathlessness?
(Please select 1 option)
Acute epiglottitis
Angio-oedema

This is the correct answer

Asthma
Pneumothorax
Pulmonary embolism

Incorrect answer selected

The history of noisy breathing is suggestive of stridor, which can be triggered by an allergic
reaction in an otherwise well adolescent, and the history of atopy is supportive of a diagnosis
of angio-oedema. Similarly, the rather abrupt history is again suggestive.
The main differential diagnosis is asthma where one would expect expiratory wheeze;
however the silent chest is an ominous feature in acute severe asthma.

Work Smart
Time taken: 04:33

Theme:Vaginal bleeding in pregnancy/puerperium


A

Abruption

Cancer of the cervix

Coagulation failure

Ectopic pregnancy

Hydatidiform mole

Incomplete miscarriage

Missed miscarriage

Placenta praevia

Retained placenta

Sepsis of retained products of conception

Severe thyrotoxicosis

Threatened abortion

Urethral chemotaxis

Uterine fibroids

In each of the following statements, select the most likely cause of vaginal bleeding.
A 16-year-old schoolgirl who recently had a surgical termination of pregnancy was re-admitted
with abnormal vaginal bleeding and severe lower abdominal pain and fever. Her haemoglobin
was 9 gm/dl and her white cell count was 20,000/mcl.

Incorrect - The correct answer is Sepsis of retained products of conception


The patient's fever and raised white cell count indicate infection and the abnormal
bleeding/anaemia indicate a septic miscarriage following a failed therapeutic abortion or
sepsis of retained products of conception (occurs in approximately 3.6%). Most common
organisms are Escherichia coli, Bacteroides, Streptococci (anaerobic and aerobic) and
Clostridium perfringens.

An Asian woman who is G3P1+1 was admitted with a history of excessive vomiting, vaginal
bleeding at 10 weeks gestation and minimal abdominal pain. Abdominal examination indicated a

soft uterus, 16 weeks' gestation, and doughy consistency. An ultrasound scan showed no fetal
parts, but a snowy pattern. The hcG was higher than the value for 10 weeks' gestation.

Incorrect - The correct answer is Hydatidiform mole


A raised hcG, which is a known cause of hyperemesis, should always raise the suspicion of
a molar pregnancy. The ultrasound appearances of echoes resembling a snowstorm is
characteristic of hydatidiform mole.

An African woman who had been investigated for 10 years primary infertility was admitted with a
history of severe onset lower abdominal pain followed by slight bleeding per vaginum. Her
menses was two days overdue. Her blood pressure was 80/50 mmHg, pulse 146/m and thready,
haemoglobin 7 g/dl. Pregnancy test was positive and ultrasound excluded any evidence of a
fetus in the uterine cavity.

Incorrect - The correct answer is Ectopic pregnancy


A positive pregnancy test in the absence of an intrauterine pregnancy on ultrasound should
always arouse suspicion of an ectopic pregnancy, which classically presents with
haemodynamic compromise. The patient's infertility is a clue to possible tubal pathology.
Bleeding per vaginum is often slight in ectopic pregnancy, and does not account for the
severe anaemia. Ultrasound examination in this case is likely to show copious free fluid.

A hypertensive, haemodynamically stable, 36 weeks pregnant woman was admitted with a


history of painless vaginal bleeding. She had a similar bleed at 20 weeks' gestation. On
abdominal examination, the uterus was consistent with her gestational age, soft and non- tender.
Fetal lie was transverse and fetal heart was audible.

Incorrect - The correct answer is Placenta praevia


Placenta praevia classically presents with a painless bleed, occurring modally at 34 weeks.
Bleeding in the second trimester can be a feature. The lie is transverse because of the
position of the placenta. A soft non-tender uterus excludes major placental abruption.

A 33-year-old woman G5P4 presented with sudden abdominal pain and slight vaginal bleeding
following a car crash. She was pale, sweating, hypertensive and her pulse rate was 120 bpm.
The uterus was tender and hard, and fetal parts were difficult to define. CTG showed signs of
fetal distress.

Incorrect - The correct answer is Abruption


Any trauma can cause shearing forces on the placenta and lead to abruption or accidental
haemorrhage. The uterus is characteristically tender and hard, and fetal parts difficult to
palpate. Separation of the placenta leads to fetal compromise. Bleeding may be concealed
and can lead to shock.

Work Smart
Time taken: 04:41

Theme:Overdosages/ poisoning
A

Digoxin

Iron

Organophosphates

Paracetamol

Paraquat

Salicylate

Sodium hypochlorite (bleach)

Tricyclics

For each patient below, choose from the given list the single most likely causative agent
responsible for their presentation:
A 7-year-old child with nausea vomiting and features of heart block.

Correct
Digoxin toxicity. Features include nausea, vomiting, impaired cognition and arrhythmias.
Treatment involves correction of hypokalaemia; digoxin specific antibody fragments are
indicated to treat serious arrhythmias.

A 45-year-old farmer presents with excessive sweating, abdominal pain and diarrhoea.

Incorrect - The correct answer is Organophosphates


Organophosphate insecticides inactivate cholinesterase resulting in increased cholinergic
symptoms of salivation, lacrimation, small pupils, sweating, urination and diarrhoea, coma
and respiratory failure. Treatment is with the anticholinergic atropine.

A 17-year-old girl is admitted with hyperventilation and is noted to be in metabolic acidosis.

Incorrect - The correct answer is Salicylate


Salicylate (present in aspirin preparations) causes hyperventilation which may result in a
respiratory alkalosis. Massive overdose may cause a metabolic acidosis.

A 32-year-old farm labourer with sore mouth, pulmonary oedema and renal failure.

Incorrect - The correct answer is Paraquat


Paraquat is found in weed-killers. This causes diarrhoea and vomiting, painful oral ulcers,
alveolitis (pulmonary oedema)and renal failure.

A 16-year-old girl presents with right upper quadrant pain and has deranged liver function tests.

Incorrect - The correct answer is Paracetamol


Paracetamol poisoning may cause vomiting, right upper quadrant pain. Later liver failure with
encephalopathy and renal failure may occur. Treatment with N-acetyl cysteine (NAC) is
given according to a standard nomogram. NAC may be useful up to 36 hours following
ingestion. The occasional patient may require liver transplantation.

Work Smart
Time taken: 04:41

An 18-year-old male with sickle cell disease presents with severe abdominal pain. His blood
pressure is 180/100 mmHg, heart rate 110 bpm, and temperature 38.0C.
What is your first action?

(Please select 1 option)


AXR
CXR
IV benzylpenicillin 2.4 g
IV normal saline

This is the correct answer

Urgent surgical consult

Incorrect answer selected

Intravenous fluids and analgesia, usually with opiates, are the cornerstones of the
management of sickle cell crisis.
Antibiotics should then be considered - specifically to cover Haemophilus influenzae type b,
Mycoplasma pneumoniae and Pneumococcus, with, for example, ceftriaxone, erythromycin
and cefuroxime respectively.
One should also consider that these patients may also develop appendicitis, like any other
young patient, so a surgical consult may be required as appropriate.
Approximately 8-10% of the African population carry the sickle cell gene. Homozygous (SS)
sickle disease occurs in about 0.15% of patients of African origin.
Nowadays the prognosis of sickle cell disease is good, with approximately 50% of patients
surviving beyond the fifth decade.

Work Smart
Time taken: 04:45

An 18-year-old with cerebral palsy is admitted after a respiratory arrest having been
intubated by paramedics. Nobody can gain intravenous access as the patient is too shut
down. A femoral line is not possible due to contractures. You do not have the experience to
perform central venous cannulation.
Which of the following is the best option for administering intravenous fluids/emergency
drugs in this situation of inability to gain venous access?
(Please select 1 option)
Down the endotracheal tube
Intramuscular
Intraosseous

This is the correct answer

Nasogastric
Subcutaneous

Incorrect answer selected

Nasogastric, IM and subcutaneous are too slow and unreliable for emergency situations
(although in cardiac arrest the endotracheal route is recognised). Venous cut down is a
possibility but requires skill in the procedure.
Intraosseous is still perfectly viable in the adult patient 2 cm below the tibial tuberosity on the
antero-medial side or 2 cm proximal to the medial malleolus.

Work Smart
Time taken: 04:50

A 25-year-old female with a history of palpitations is admitted with a pulse of 160 beats per
min, blood pressure 105/80 mmHg and a respiratory rate 26/min. She is a awake, alert and
oriented and is not distressed. Her electrocardiogram shows a regular QRS complex of
0.10s duration.
Which of the following is the most appropriate therapy in this patient?
(Please select 1 option)
Adenosine 3 mg
DC cardioversion
Diltiazem 15mg
Vagal manoeuvres
Verapamil 2.5mg

This is the correct answer


Incorrect answer selected

This is a narrow complex tachycardia with normal blood pressure and an undistressed
patient allows the trial of vagal manoeuvres before drug therapy.
Carotid sinus massage or valsalva is effective in 25% of cases.

Work Smart
Time taken: 04:58

Theme:Blood gas analysis


A

Metabolic acidosis - acute, normal oxygenation

Metabolic acidosis - acute with hypoxaemia

Metabolic acidosis - compensated, normal oxygenation

Metabolic alkalosis - acute, normal oxygenation

Metabolic alkalosis - acute with hypoxaemia

Metabolic alkalosis - compensated, normal oxygenation

Normal blood gases

Normal ventilation with hypoxaemia

Respiratory failure - acute

Respiratory failure - compensated, normal oxygenation

Respiratory failure - compensated with hypoxaemia

For each of the following blood gas results, select the most appropriate designation from the
list of options.
The line labelled oxygen concentration represents the concentration of oxygen that each
individual is inhaling.
Normal values are:
PaO2

12.0-14.7 kPa (90-110 mmHg)

PaCO2

4.5-6.0 kPa (34-45 mmHg)

pH

7.36-7.44

H + molarity

35-45 nmol/L

Bicarbonate

24-30 mmol/L

pO2

10.0 kPa (80 mmHg)

pCO2

5.8k Pa (44 mmHg)

pH

7.24

H+ molarity

57 nmol/L

Bicarbonate

18 mmol/L

Base Excess

7 mmol/L

Oxygen concentration

40%

Correct
The low pH with low bicarbonate indicates a metabolic acidosis, and the low normal O2 on
inspired [O2] indicates hypoxaemia. This may occur with poisoning and associated
respiratory failure.

pO2

6.7 kPa (50 mmHg)

pCO2

10.1 kPa (76 mmHg)

pH

7.38

H+ molarity

42 nmol/L

Bicarbonate

42.4 mmol/L

Base excess

+14 mmol/L

Oxygen concentration

21%

Incorrect - The correct answer is Respiratory failure - compensated with hypoxaemia


This is the typical feature of a corrected respiratory failure with hypoxaemia. The PaO2 is low,
the PaCO2 is high, with raised bicarbonate indicating renal correction and a high base
excess.

pO2

14.3 kPa (108 mmHg)

pCO2

6.3 kPa (48 mmHg)

pH

7.52

H+ molarity

30 nmol/L

Bicarbonate

39 mmol/L

Base excess

+14 mmol/L

Oxygen concentration

30%

Incorrect - The correct answer is Metabolic alkalosis - acute, normal oxygenation


Metabolic alkalosis, for example in a patient with severe vomiting, is characterised by a high
pH, high bicarbonate and a raised base excess.

pO2

4.7 kPa (35 mmHg)

pCO2

12.7 kPa (95 mmHg)

pH

7.12

H+ molarity

76 nmol/L

Bicarbonate

29.5 mmol/L

Base excess

4mmol/L

Oxygen concentration

21%

Incorrect - The correct answer is Respiratory failure - acute


There is a type 2 respiratory failure with hypoxia, hypercapnia and acidosis with a normal
bicarbonate, indicating an acute respiratory failure, for example in severe asthma or
pneumonia.

pO2

16.3 kPa (122 mmHg)

pCO2

7.5 kPa (56 mmHg)

pH

7.26

H+ molarity

55 nmol/L

Bicarbonate

24.1 mmol/L

Base excess

2 mmol/L

Oxygen concentration

75%

Incorrect - The correct answer is Respiratory failure - acute


There is an acidosis with a normal bicarbonate, and hypercapnia, on high flow O2. This is the
picture of acute respiratory failure often found in patients with chronic obstructive airways
disease who have lost their hypoxic drive and have been given high concentrations of O2.

Work Smart
Time taken: 05:01

You are on call for hospital at night and are urgently called to a patient on the ward who is
choking on a piece of steak visible in his oropharynx. He is in extremis with saturations of
87%.
Which of the following is the most appropriate immediate management for this patient?
(Please select 1 option)
Cricothyroidotomy
Finger sweep
Heimlich manoeuvre

This is the correct answer

High flow oxygen


Removal with forceps

Incorrect answer selected

A finger sweep is more likely to push the obstruction further into the airway and is no longer
advocated.

High flow oxygen is the breathing part of A, B, C, and the airway is not clear.
Nasopharyngeal airways will not help in this situation.
Removal with forceps is potentially hazardous.
A Heimlich manoeuvre should be performed with the possibility of cricothyroidotomy in mind
immediately thereafter if this procedure fails.
In some countries, notably Australia, this procedure is not encouraged, mainly because
overvigorous use can cause internal injury. However it is still recommended in the US and
the UK .

Work Smart
Time taken: 05:15

Theme:Blood gas analysis


A

Metabolic acidosis - acute, normal oxygenation

Metabolic acidosis - acute with hypoxaemia

Metabolic acidosis - compensated, normal oxygenation

Metabolic alkalosis - acute, normal oxygenation

Metabolic alkalosis - acute with hypoxaemia

Metabolic alkalosis - compensated, normal oxygenation

Normal blood gases

Normal ventilation with hypoxaemia

Respiratory failure - acute

Respiratory failure - compensated, normal oxygenation

Respiratory failure - compensated with hypoxaemia

For each of the following blood gas results select the most appropriate designation from the
list of options.

The line labelled 'oxygen concentration' represents the concentration of oxygen that each
individual is inhaling.
Normal values
PaO2

12.0-14.7 kPa

PaO2

90-110 mmHg

PaCO2

4.5-6.0 kPa

PaCO2

34-45 mmHg

pH

7.36-7.44

H+ molarity

35-45 nmol/l

Bicarbonate

24-30 mmol/l

pO2

12.9 kPa (97 mmHg)

pCO2

5.8 kPa (44 mmHg)

pH

7.24

H+ molarity

57 nmol/l

Bicarbonate

18 mmol/l

Base excess

-7 mmol/l

Oxygen concentration

40%

Incorrect - The correct answer is Metabolic acidosis - acute with hypoxaemia


The low pH with low bicarbonate indicates a metabolic acidosis, and the low normal O2 on
inspired [O2] indicates hypoxaemia.

pO2

6.7 kPa (50 mmHg)

pCO2

10.1 kPa (76 mmHg)

pH

7.38

H+ molarity

42 nmol/l

Bicarbonate

42.4 mmol/l

Base excess

+14 mmol/l

Oxygen concentration

21%

Incorrect - The correct answer is Respiratory failure - compensated with hypoxaemia


This is the typical feature of a corrected respiratory failure with hypoxaemia. The PaO2 is low,
the PaCO2 is high, with raised bicarbonate indicating renal correction and a high base
excess.

pO2

14.3 kPa (108 mmHg)

pCO2

6.3 kPa (48 mmHg)

pH

7.52

H+ molarity

30 nmol/l

Bicarbonate

39 mmol/l

Base excess

+14 mmol/l

Oxygen concentration

30%

Incorrect - The correct answer is Metabolic alkalosis - acute, normal oxygenation


Metabolic alkalosis, for example in a patient with severe vomiting, is characterised by a high pH, high
bicarbonate and a raised base excess. In an effort to compensate, the respiratory drive decreases
and hence the PCO2 begins to rise.

pO2

4.7 kPa (35 mmHg)

pCO2

12.7 kPa (95 mmHg)

pH

7.12

H+ molarity

76 nmol/l

Bicarbonate

29.5 mmol/l

Base excess

-4 mmol/l

Oxygen concentration

21%

Incorrect - The correct answer is Respiratory failure - acute


Hypoxia, hypercapnia and acidosis with a normal bicarbonate, indicate an acute respiratory
failure, for example in massive pulmonary embolism.

pO2

16.3 kPa (122 mmHg)

pCO2

7.5 kPa (56 mmHg)

pH

7.26

H+ molarity

55 nmol/l

Bicarbonate

24.1 mmol/l

Base excess

-2 mmol/l

Oxygen concentration

75%

Incorrect - The correct answer is Respiratory failure - acute


Here is an acidosis with a normal bicarbonate, and hypercapnia, on high flow O2. This is the
picture of acute respiratory failure often found in patients with chronic obstructive airway
disease who have lost their hypoxic drive and have been given high concentrations of O2.

Work Smart
Time taken: 05:22

Theme:Abdominal pain
A

Acute cystitis

Addison's disease

Appendicitis

Cancer of the colon

Chronic inflammatory bowel disease

Diverticular disease

Ectopic pregnancy

Endometriosis

Inguinal hernia

Lead poisoning

Pelvic inflammatory disease

Primary spasmodic dysmenorrhoea

Pseudo-obstruction

Ruptured follicular cyst

Toxic megacolon

Urinary retention

Vesical calculus

Volvulus

Each of the subjects below is a woman presenting with lower abdominal pain.
Select the most likely diagnosis from the list of options.
A 23-year-old woman presents with a three day history of low abdominal pain, frequency of
micturition and a yellow, offensive vaginal discharge. Her bowel actions are regular and her
weight is steady. Her last period was three weeks previously and she is on no contraception.

Incorrect - The correct answer is Pelvic inflammatory disease


Pelvic inflammatory disease is associated with a malodorous, green or yellow discharge. It
may be associated with STDs such as Chlamydia but may occur in a sexually inactive
female.

A 24-year-old woman presents with a four month history of right-sided lower abdominal pain,
watery diarrhoea and a loss of weight of 6 kg. Her periods are regular taking the oral
contraceptive pill.

Incorrect - The correct answer is Chronic inflammatory bowel disease


The most likely answer in this case is chronic inflammatory bowel disease given the weight
loss, abdominal pain and diarrhoea. Addison's disease would present insidiously, and
typically without bowel symptoms.

A 65-year-old with a ten year history of constipation and of left-sided lower abdominal pain which
is worse on defecation. Her weight is steady and she has normal micturition, She has had no
periods for 15 years.

Incorrect - The correct answer is Diverticular disease


In a patient of this age, diverticular disease is common and would present in this fashion.
The fact that weight loss is not present suggests that malignancy is not present.

A 30-year-old woman presents with a six hour history of severe pain in the lower abdomen. She
has had a normal bowel motion this morning and her weight is steady. Her last period was six
weeks ago and she has an IUCD for contraception. Her BP is 90/60 mmHg and HR 100 bpm.

Incorrect - The correct answer is Ectopic pregnancy


This patient has a short, acute history of abdominal pain and has features of early shock.
She has missed her last period and has normal bowel motions, with no weight loss. The
IUCD is associated with a small risk of ectopic pregnancy, and this is the most likely
diagnosis from the above list.

A 15-year-old girl presents with agonising pain in the epigastrium and right lower abdomen for
four hours. She has vomited once but not had a bowel motion since the previous day. Her weight
is steady. Her periods started 12 months ago and are still irregular, but the last was three weeks
ago. She denies being sexually active.

Incorrect - The correct answer is Appendicitis


This patient has an acute history of right lower abdominal pain, vomiting and relative
constipation. Appendicitis is relatively common at this age, and is the most likely diagnosis in
this scenario.

Work Smart
Time taken: 05:30

Theme:Upper gastrointestinal haemorrhage

Acute erosive gastritis

Aorto-enteric fistula

Duodenal lymphoma

Gastric leiomyoma

Mallory-Weiss tear

Oesophageal varices

Oesophagitis

Peptic ulcer

For each case below, choose the single most appropriate diagnosis from the given list of
options.
Each option may be used once, more than once, or not at all.
A 47-year-old man known to abuse alcohol presents to the Emergency department with a
profuse haematemesis. He is found on examination to be jaundiced, mildly confused, with
ascites and a liver flap. His serum albumin is markedly low.

Incorrect - The correct answer is Oesophageal varices


The alcohol abuse and profuse haematemesis are suggestive of varices. Plus, this patient
has evidence of portal hypertension.

A 21-year-old student is brought into the Emergency department with severe vomiting and
diarrhoea. He has just returned from a holiday in Sri Lanka and is dehydrated. He continues to
vomit and then complains of severe retrosternal pain and has a haematemesis.

Incorrect - The correct answer is Mallory-Weiss tear


Typical history of recurrent vomiting, then blood.

A warden in a block of flats is called to see a 62-year-old with a massive haematemesis. He has
a midline laparotomy scar from an aortic aneurysm repair and is vomiting large quantities of fresh
blood.

Correct
The midline laparotomy and massive haematemesis after AAA repair are highly suggestive
of aorto-enteric fistula.

A possibility is peptic ulceration/stress ulcer but, in the circumstances, the examiner is


probably seeking the former answer.

A 70-year-old man is taken to theatre with a massive upper gastrointestinal (GI) bleed. At
laparotomy he is found to have a large mass in the gastric body which is rubbery in nature and is
bleeding profusely.

Incorrect - The correct answer is Gastric leiomyoma


The rubbery large mass suggests leiomyoma. These are prone to haemorrhage.

A 42-year-old man who was previously fit and well presents with a sudden onset of
haematemesis. He has noticed melaena stool during the day and is on no medication.

Incorrect - The correct answer is Peptic ulcer


This patient has been previously fit and well and taking no medication. Of the options offered
it seems most likely that this is due to an acute peptic ulceration.

Patients with severe upper GI haemorrhage require resuscitation with respect to their airway,
breathing, and circulation. A history should be gained contemporaneously. The underlying
cause should then be treated.
Bleeding ulcers are injected with adrenalin or underrun; varices may be injected or banded.

Work Smart
Time taken: 05:36

Theme:Gastrointestinal Symptoms

Angiodysplasia

Duodenal ulcer

Caecal carcinoma

Crohn's disease

Mallory-Weiss tear

Oesophageal varices

Rectal carcinoma

Sigmoid carcinoma

Sigmoid diverticular disease

For each of the patients below, select the most likely diagnosis from the listed options:
A 56-year-old man presents with anaemia and weight loss. Examination reveals a mass in the
right iliac fossa and hepatomegaly.

Incorrect - The correct answer is Caecal carcinoma


This patient has presented with anaemia which suggests chronic blood loss. The most likely
diagnosis is that of a caecal carcinoma which is therefore associated with a mass in the right
iliac fossa and weight loss. Hepatomegaly suggests metastatic disease.

A 50-year-old alcoholic presents with melaena. On examination the patient is drowsy and
hypotensive. Examination of the abdomen shows splenomegaly.

Incorrect - The correct answer is Oesophageal varices


An alcoholic person with haematemesis and melaena could have a duodenal ulcer or
gastritis or oesophageal varices. The presence of splenomegaly however, suggests that he
probably has portal hypertension and oesophageal varices are a possibility. Drowsiness
implies hepatic encephalopathy precipitated by the bleeding.

A 58-year-old man is admitted with acute onset of left iliac fossa pain. Recently he has noticed he
has had some vague abdominal pain and felt more constipated. On examination he is pyrexial

and has localised peritonism in the left iliac fossa. His abdomen is distended. Investigations show
a Hb of 12 g/dL, WCC of 18 109/L.

Incorrect - The correct answer is Sigmoid diverticular disease


The presence of pyrexia and localised peritonism in the left iliac fossa suggests the
presence of inflammation. The history suggests a perforated or severely inflamed sigmoid
colon. The diagnosis is most likely to be diverticulitis with a localised perforation. A
perforated sigmoid tumour must also be considered but is less common.

A 77-year-old man presents with his fourth episode of acute rectal bleeding. The blood is a
mixture of fresh blood and clots. On this occasion the bleeding has been severe enough to
require a 4 unit blood transfusion. A barium enema is undertaken and is normal.

Correct
The differential diagnosis is diverticular disease or angiodysplasia. Both can produce a
significant gastrointestinal blood loss which is painless. In contrast a sigmoid carcinoma
would not usually bleed enough to require a transfusion but would be associated with
chronic blood loss causing anaemia. Angiodysplasia is only diagnosed by colonoscopy.
Diverticulae would be seen on a barium enema.

Work Smart
Time taken: 05:39

A 43-year-old woman attends casualty with breathlessness.


Clinical signs suggest a chest infection. The only medication she has been taking is an over
the counter preparation for dyspepsia.
Blood gases show:
pH

7.56

(7.36-7.44)

pO2

11.0 kPa

(11.3-12.6)

pCO2

4.8 kPa

(4.7-6.0)

O2Saturation

95%

HCO3

3 8mmol/L

(20-28)

Which of the following is the likely culprit?


(Please select 1 option)
Calcium carbonate

This is the correct answer

Carbenoxolone
Magnesium trisilicate
Ranitidine
Sucralfate

Incorrect answer selected

This patient has a metabolic alkalosis with a raised pH and serum bicarbonate. Both
carbenoxolone and calcium carbonate can cause this but carbenoxolone is only available on
prescription.

Work Smart
Time taken: 05:46

A 24-year-old female is admitted with palpitations. Her pulse is 220 beats/min, blood
pressure 70/50 mmHg and she has a respiratory rate 32/min.
She is awake, alert and oriented but dyspnoeic. Her electrocardiogram shows a regular
rhythm with QRS complex width of 0.11s.
What is the most appropriate therapy for this patient?
(Please select 1 option)
Adenosine 6 mg/6 mg/12 mg
Amiodarone 300 mg
Atenolol 50 mg
Direct current cardioversion
Verapamil 10 mg

This is the correct answer

Incorrect answer selected

This is highly likely to be a narrow complex tachycardia.


Strictly speaking, as this patient is showing signs of haemodynamic decompromise (that is,
systolic blood pressure less than 90) she should be immediately DC cardioverted under
sedation/anaesthesia.

In practice, most people would try adenosine first whilst organising a cardioversion.

Work Smart
Time taken: 05:50

A 27-year-old female presents to the surgical intake with abdominal pain and a five day
history of vomiting.
Over the last three months she has also been aware of a 6 kg weight loss.
On examination, she is pale, has a temperature of 38.5C, blood pressure of 90/60 mmHg
and pulse rate of 130 in sinus rhythm. The chest is clear on auscultation but she has a
diffusely tender abdomen with no guarding. Her BM reading is 2.5.
Initial biochemistry is as follows:
Sodium

124 mmol/l

(137-144)

Potassium

6.0 mmol/l

(3.5-4.9)

Urea

7.5 mmol/l

(2.5-7.5)

Creatinine

78 mol/l

(60-110)

Glucose

2.0 mmol/l

(3.0-6.0)

What is the probable diagnosis?


(Please select 1 option)
Abdominal migraine
Acute appendicitis
Acute cholecystitis
Addison's disease
Insulinoma

This is the correct answer

Incorrect answer selected

This patient has clinical features of hypoadrenal crisis with abdominal pain, vomiting and
shock with hypoglycaemia, hyponatraemia and hyperkalaemia. In the United Kingdom this is
commonly due to autoimmune destruction of the adrenal glands (Addison's disease).
Rarer causes are TB, HIV adrenal haemorrhage or anterior pituitary disease.

Patients classically present with weight loss, abdominal pain, lethargy and nausea and
vomiting. Addison's disease is also associated with oral pigmentation (due to excess ACTH
and therefore MSH), and other autoimmune disease (including thyroid disease and vitiligo).
Patients such as this need emergency fluid resuscitation, steroid administration (prior to this
urgent cortisol measurement), and careful search for occult infection.
None of the other options explain the biochemical findings.

Work Smart
Time taken: 05:56

A 78-year-old male is brought to the Emergency department and has a witnessed seizure in
the resuscitation room.
His blood glucose is recorded as 1.0 mmol/l. He does not have diabetes, nor other
significant medical history.
He is given 50 ml of 50% dextrose and he slowly recovers over the next one hour. A serum
cortisol concentration later returns as 800 nmol/l (120-600).
Which of the following would be the most relevant investigation for this man?
(Please select 1 option)
Chest x ray
CT head scan
Electrocardiogram
Prolonged 72 hour fast
Short Synacthen test

This is the correct answer


Incorrect answer selected

The historical and biochemical evidence here suggests a diagnosis of spontaneous


hypoglycaemia and the most likely cause would be an insulinoma. However, one would wish
to exclude possible drug administration and although not mentioned here, a sulphonylurea
screen should be undertaken.
He has presented with symptomatic hypoglycaemia, is not diabetic and therefore should not
have received insulin or a sulphonylurea.
There is nothing to suggest alcohol or drug misuse. Similarly, there is nothing to suggest
sepsis.

However, to prove a diagnosis of spontaneous hypoglycaemia a prolonged fast is required


and, should he develop hypoglycaemia, measurement of insulin and C peptide will be
needed to confirm the diagnosis.
The appropriate cortisol response during his hypoglycaemic episode (cortisol 800) excludes
hypoadrenalism.

Work Smart
Time taken: 05:59

A 62-year-old male with a history of ischaemic heart disease is admitted with chest pain of
13 hours duration and some ST depression inferolaterally.
Which of the following would be the most appropriate treatment option for this patient?
(Please select 1 option)
IV diamorphine 10 mg

Oral aspirin 600 mg

Oral isosobide mononitrate 30 mg

Oxygen 100%

Subcutaneous fondaparinux 2.5 mg

Correct

The treatment approach in this patient with acute coronary syndrome is:
Aspirin 300 mg
Try some glyceryl trinitrate (GTN).

If this fails then:


Isosorbitdinitrate (ISDN) intravenously
Diamorphine (or morphine) 2.5 mg
4 L oxygen, and

fondaparinux - a subcutaneous anticoagulant.

Work Smart
Time taken: 06:04

A 66-year-old man with insulin-dependent diabetes given ibuprofen for a knee injury is
admitted with palpitations.
His electrocardiogram (ECG) shows a rate of 105 beats per minute, with absent P waves
and tall T waves.
His urea and electrolytes show:
Sodium

132 mmol/L

(137-144)

Potassium

6.4 mmol/L

(3.5-4.9)

Urea

11 mmol/L

(2.5-7.5)

Creatinine

180 mol/L

(60-110)

In this scenario, which of the following is the most appropriate immediate management?
(Please select 1 option)
Calcium chloride 10 mmol IV

This is the correct answer

Calcium resonium orally


Dextrose 50 mls 50% with 10 units insulin
Dialysis
Furosemide 1 mg/kg IV

Incorrect answer selected

The ECG suggests cardiotoxicity related to hyperkalaemia and the history of palpitations is
suggestive of arrhythmias.
Therefore cardio protection with calcium chloride or gluconate should be first priority and
lowering potassium levels immediately thereafter.
Calcium antagonises the effects of hyperkalaemia on a cellular level by a number of
mechanisms. These all return myocyte excitability to normal thereby reducing the risk of
arrhythmias. Calcium gluconate is the preferred preparation, but calcium chloride can be
used. Caution is needed if the patient is taking digoxin, as hypercalcaemia can potentiate its

toxicity. The effects of intravenous calcium occur within one to three minutes but last for only
30-60 minutes, and therefore more definitive treatment is needed to lower potassium levels.
After calcium is given, treatment is required to shift potassium intracellularly. Insulin is most
commonly used, which stimulates the Na-K ATPase pump. The effect is seen within 10-20
minutes and usually decreases potassium levels by 0.6-1mEq/L. Salbutamol can also
increase the action of the Na-K ATPase pump.
Sodium bicarbonate infusion can shift potassium intracellularly by increasing blood pH, but
its use is controversial. It is therefore reserved for cases of severe acidosis, or where there is
another indication for its use (for example, TCA overdose).
If the above treatments fail, and the cause of hyperkalaemia cannot be treated, then
potassium may need to be removed from the body. The most efficient way to do this is with
haemodialysis. This is only done in resistant cases, or in patients who are already on
haemodialysis. For most patients, treatment with an exchange resin such as sodium
polystyrene sulphonate is more appropriate.

Work Smart
Time taken: 06:10

A 67-year-old man with insulin-dependent diabetes with a broad complex pulseless


tachyarrhythmia (with a protected airway) has just been defibrillated for the third time without
return of cardiac output. CPR is immediately resumed and adrenaline administered.
Which of the following is the next step in the management of the arrest?
(Please select 1 option)
Adrenaline 1 mg
Amiodarone 300 mg

This is the correct answer

DC shock
Lidocaine 100 mg
Removal of oxygen and then DC shock

Incorrect answer selected

The next step is amiodarone 300 mg intravenously (i.e. given after the third shock). If
amiodarone is not available lidocaine is a suitable alternative.

Work Smart

Time taken: 06:16

A 22-year-old female is admitted very distressed and short of breath.


Examination reveals a respiratory rate of 35/min, a pulse of 120 beats per minute, a blood
pressure 110/70 mmHg, oxygen saturations of 90% and a peak expiratory flow rate <50%
predicted.
The emergency medical services have administered salbutamol 5 mg (twice) and face mask
oxygen.
Which of the following is the most appropriate next action in this patient?
(Please select 1 option)
Arterial blood gas analysis
Intensive care referral
Oxygen 35%
Prednisolone 40 mg
Salbutamol 5 mg and ipratropium bromide 0.5 mg

Correct

This patient fits the criteria for acute severe asthma. In such cases 2-agonists should be
administered as soon as possible, preferably nebulised driven by oxygen. Repeat doses
should be given at 15-30 minute intervals, or continuous nebulisation can be used where
there is inadequate response to bolus therapy. Nebulised ipratropium bromide should be
added for patients with acute severe or life threatening asthma, or those with a poor initial
response. Its addition produces significantly greater bronchodilation than a 2-agonist alone.
Oxygen should be given to maintain saturations at 94-98%. Patients with saturations less
than 92% on air should have an ABG to exclude hypercapnia. However, starting treatment
should not be delayed to do the ABG. Initially high-flow oxygen is used, and then weaned to
maintain adequate saturations. Unless you suspect COPD there is not a need to be cautious
with oxygen therapy.
Steroids reduce mortality, relapses, subsequent hospital admission and requirement for 2agonists1. The earlier they are given in the attack, the better the outcome. A dose of 40-50
mg should therefore be given once oxygen and nebuliser therapy has been established. This
should be continued for five days, or until recovery, and can then be stopped abruptly unless
the patient has taken long term oral corticosteroids.
Failure to respond to the above treatment steps may warrant the use of intravenous
magnesium sulphate and aminophylline. However, these should not be used without
discussion with your senior colleagues.
Intensive care is indicated for patients with severe acute or life threatening asthma who are
failing to respond to therapy. Consider it in patients with deteriorating peak flow, persisting or
worsening hypoxia, hypercapnia, acidosis, exhaustion or altered conscious state. All patients
who are transferred to an intensive care unit should be accompanied by a doctor who can
intubate if necessary.

As an aside, chest radiographs are not indicated unless you suspect pneumothorax or
consolidation, or there is life-threatening asthma, a failure to respond to treatment or a need
for ventilation.

Work Smart
Time taken: 06:20

A 22-year-old male is admitted wheezing with a respiratory rate of 35/min, a pulse of 120
beats per min, blood pressure 110/70 mmHg, peak expiratory flow rate <50% predicted.
The emergency medical services have administered salbutamol 5 mg (twice), ipratroprium
0.5 mg and face mask oxygen.
His arterial blood gas reveals:
pH

7.42

(7.36-7.44)

PaCO2

5.0 kPa

(4.7-6.0)

PaO2

22 kPa

(11.3-12.6)

Base excess

-2 mmol/l

(+/-2)

SpO2

98

Which of the following is the most appropriate action for this man?
(Please select 1 option)
Chest x ray

Intensive care referral

Ipratropium

Magnesium 1-2 g

Oxygen 35 %

This is the correct answer

Incorrect answer selected

This patient fits the criteria for life threatening asthma.


A normal PaCO2 in an asthmatic is a warning of impending respiratory failure as the patient
becomes too tired to ventilate adequately.
Initial treatment has been given: 2-agonists should be administered as soon as possible,
preferably nebulised driven by oxygen. Repeat doses should be given at 15-30 minute
intervals, or continuous nebulisation can be used where there is inadequate response to
bolus therapy.
Nebulised ipratropium bromide should be added for patients with acute severe or life
threatening asthma, or those with a poor initial response. Its addition produces significantly
greater bronchodilation than a 2-agonist alone.
Oxygen should be given to maintain saturations at 94-98%. Patients with saturations less
than 92% on air should have an ABG to exclude hypercapnia. However, starting treatment
should not be delayed to do the ABG. Initially high-flow oxygen is used, and then weaned to
maintain adequate saturations. Unless you suspect COPD there is no need to be cautious
with oxygen therapy.
This gentleman is receiving adequate ipratropium and oxygen, and repeating/increasing
these are unlikely to help the situation markedly.
Failure to respond to the above treatment steps warrants the use of intravenous magnesium
sulphate (or aminophylline, but this is not an option here). Magnesium has been shown to
result in bronchial smooth muscle relaxation.
Intensive care is indicated for patients with severe acute or life threatening asthma who are
failing to respond to therapy. Consider it in patients with
Deteriorating peak flow
Persisting or worsening hypoxia
Hypercapnia
Acidosis
Exhaustion or
Altered conscious state.

All patients who are transferred to an intensive care unit should be accompanied by a doctor
who can intubate if necessary. This would be considered if this gentleman fails to respond to
magnesium, and in a clinical setting you would want to discuss him with your ITU colleagues
whilst the magnesium was being given.
Chest radiographs are not indicated unless you suspect pneumothorax or consolidation, or
there is life threatening asthma, a failure to respond to treatment or a need for ventilation.

As an additional point, steroids reduce mortality, relapses, subsequent hospital admission


and requirement for 2-agonists1. The earlier they are given in the attack, the better the
outcome.
A dose of 40-50 mg should therefore be given once oxygen and nebuliser therapy has been
established. This should be continued for five days, or until recovery, and can then be
stopped abruptly unless the patient has taken long term oral corticosteroids.

Work Smart
Time taken: 06:23

A 59-year-old man is admitted with chest pain of eight hours duration and has ST elevation
in the inferior leads on his admission ECG.
An electrocardiogram from a previous clinic visit shows sinus rhythm two months ago. He
has insulin-dependent diabetes mellitus and chronic renal failure.
Investigations reveal:
Fasting plasma glucose

7.4 mmol/L

(3.0-6.0)

Sodium

137 mmol/L

(137-144)

Potassium

4.4 mmol/L

(3.5-4.9)

Urea

10 mmol/L

(2.5-7.5)

Creatinine

200 mol/L

(60-110)

Which of the following which represent an absolute contraindication to the use of


thrombolysis?
(Please select 1 option)
Allergy to penicillin

Gastrointestinal bleeding in last three months

History of haemorrhagic stroke

This is the correct answer

Ischaemic stroke 12 months ago

On warfarin therapy

Incorrect answer selected

Absolute contraindications to thrombolysis include:


Previous haemorrhagic stroke
Ischaemic stroke in last six months
Central nervous system damage or neoplasm
Within three weeks of major surgery, head injury or major trauma
Active internal bleeding (menses excluded) or gastrointestinal bleeding within the past
month
Known or suspected aortic dissection
Known bleeding disorder
Proliferative diabetic retinopathy.

Allergy and oral anticoagulants are relative contraindications.

Work Smart
Time taken: 06:27

A 66-year-old man with insulin-dependent diabetes with a treated potassium of 5.4 mmol/l
(3.5-5) and palpitations develops pulseless ventricular tachycardia. The anaesthetist is
supporting airway and breathing.
Which of the following is the next step in his management?
(Please select 1 option)

Adrenaline 1 mg
Amiodarone 300 mg
Cardiopulmonary resuscitation 15:2 for 2 mins
Defibrillation at 150 J biphasic
Praecordial thump

This is the correct answer

Incorrect answer selected

In the case of ventricular tachycardia and ventricular fibrillation immediate defibrillation is the
treatment of choice.
The other options come further down the algorithm and CPR is now recommended at 30:2.
A praecordial thump can be administered immediately following a witnessed cardiac arrest.

Work Smart
Time taken: 06:32

A patient has just received intravenous ceftazidime. He immediately became flushed and
wheezy, with a blood pressure of 80/40 mmHg.
Which of the following is the most appropriate immediate management for this patient?
(Please select 1 option)
Chlorphenamine 10 mg IV
Epinephrine 0.2 ml of 1:1000 IV
Epinephrine 0.5 mg IV
Epinephrine 0.5 mg IM

This is the correct answer

Hydrocortisone 100 mg IV

Incorrect answer selected

Immediate treatment of anaphylaxis includes cessation of whatever caused it.


Then give oxygen, fluids and adrenaline/epinephrine 0.5 mg intramuscularly or
subcutaneously.
Checking concentrations of adrenaline is very important especially in high pressure
situations. Intravenous adrenaline is potentially hazardous unless diluted appropriately.

#Work Smart
Time taken: 06:36

A 67-year-old woman is admitted with blackouts.


Her electrocardiogram shows ventricular escape with complete heart block. As you are
standing there she blacks out once more. Her rhythm strip shows P wave asystole.
Which of the following would be the initial immediate treatment here after airway and
breathing?
(Please select 1 option)
Adenosine 6 mg
Adrenaline 1 mg
Atropine 0.6 mg
Transcutaneous pacing
Transvenous pacing

This is the correct answer

Incorrect answer selected

Occasionally, atrial electrical activity continues in the absence of ventricular impulses. This is
referred to as P-wave asystole and may respond to electrical pacing. This can be achieved
by transvenous, transcutaneous or manual techniques. Transvenous pacing takes longer to
instigate, and transcutaneous pacing is therefore the initial choice here. Manual pacing is an
effective holding measure before more definitive pacing is instituted.
Atropine can also be used, as can adrenaline, for sustained P wave asystole but pacing is
the initial treatment of choice. All crash trollies within UK hospitals contain the equipment
required to externally pace patients.
Adenosine can induce asystole, and is only indicated in the treatment of supraventricular
tachycardias

Work Smart
Time taken: 06:40

A 25-year-old woman is admitted on the medical intake.


She is 10 weeks post partum and has been generally unwell for two weeks with malaise
sweats and anxiety.

On examination she is haemodynamically stable, and clinically euthyroid.


TFTs show the following:
Free T4

33 pmol/L

(9-23)

Free T3

8 nmol/L

(3.5-6)

TSH

<0.02 mU/L

(0.5-5)

What is the appropriate management?


(Please select 1 option)
Carbimazole 40 mg/day
Lugol's iodine
Propranolol 20 mg tds

This is the correct answer

Propylthiouracil 50 mg/tds
Radioactive iodine therapy

Incorrect answer selected

The diagnosis here is likely to be post partum thyroiditis which tends to occur within the three
months of delivery followed by a hypothyroid phase at three to six months, followed by
spontaneous recovery in one third of cases. In the remaining two-thirds, a single-phase
pattern or the reverse occurs.
Management is centred on symptomatic treatment using beta-blockers for relief of tremor or
anxiety, and observation for the development of persistent hypo- or hyperthyroidism.
Graves' disease is a less likely diagnosis based on the proximity to delivery and the absence
of any other signs to suggest Graves' ophthalmopathy, goitre and bruit.
Hashitoxicosis is a possibility but is less likely than Graves'.

Work Smart
Time taken: 06:44

A 52-year-old schoolteacher attends with weight loss and sweats. She is clinically thyrotoxic
with a diffuse goitre.
Subsequent investigations show:
Free T4

40 pmol/l

(9-23)

Free T3

9.8 nmol/l

(3.5-6)

TSH

6.1 mU/l

(0.5-5)

A repeat TFT is similar.


What is the most appropriate investigation for this patient?
(Please select 1 option)
FNA of thyroid gland
MRI scan pituitary gland

This is the correct answer

Radio-isotope uptake scan of thyroid gland


Repeat TFT checking for antibody interference
Thyroid auto antibodies

Incorrect answer selected

This patient is thyrotoxic; however, as the non-suppressed thyroid-stimulating hormone


(TSH) suggests that this is due to excessive TSH production by the pituitary gland, the
possibility of a thyrotroph adenoma must be pursued.
In primary hyperthyroidism the TSH should always be suppressed by negative feedback,
which is not the case here.
TSH-omas are indeed very rare, but the giveaway would be the normal or elevated TSH with
thyrotoxicosis

Work Smart
Time taken: 06:48

A 24-year-old law student attends with visual loss affecting the right eye. She reports a
constant headache for the last three months, and absence of menses for six months.
On examination her visual acuity in the right eye is 6/24, with slight constriction of the
temporal field in that eye but she has no other neurological deficit. She is afebrile and
haemodynamically stable.
What is the diagnosis?
(Please select 1 option)
Glaucoma

Migraine
Multiple sclerosis
Pituitary tumour
Pregnancy

This is the correct answer

Incorrect answer selected

This patient has a pituitary lesion; the history of headache and amenorrhoea suggests a
prolactinoma or non-functioning tumour. This has been complicated by involvement of the
visual pathway, which has precipitated her visual loss.
She needs urgent assessment of her pituitary function, imaging of the pituitary gland by MRI
scanning.
One of the most important investigations to perform would be a serum prolactin.
The amenorrhoea would argue against this being retrobulbar neuritis associated with MS
and similarly the peripheral visual field constriction would be unusual as a central scotoma
and fluctuating visual loss would be more typical

Work Smart
Time taken: 06:52

A 45-year-old man attends the Emergency department with symptoms suggestive of


community acquired pneumonia.
On examination he is pyrexial at 38.0C and has a respiratory rate of 32/min, with a blood
pressure of 85/55 mmHg.
Which of the following combination of features are not necessarily an indication for urgent
hospital admission?
(Please select 1 option)
BP of 85/55 mmHg and respiratory rate of 32/min

BP of 85/55 mmHg and urea of 7.5 mmol/l

Confusion and BP of 85/55 mmHg

Pyrexia of 38.0C and serum urea of 7.5 mmol/l

This is the correct answer

Respiratory rate of 32 and blood urea of 7.5 mmol/l

Incorrect answer selected

This patient has community-acquired pneumonia (CAP).


The severity of each case of CAP should be assessed using the CURB-65 tool in
conjunction with clinical judgement. Patients score 1 point for each of:
Confusion
Urea 7mmol/L
Respiratory rate 30/min
Blood pressure: systolic <90 or diastolic 60mmHg
Age 65 years

A CURB-65 score of 0 or 1 are at low risk of death, and can be treated at home if the social
circumstances are compatible. A score of 2 usually indicates inpatient treatment is required,
but hospital-supervised outpatient treatment can be considered. Patients who have a CURB65 score of 3 or more are at high risk of death. Those with scores of 4 and 5 should be
considered for treatment in a critical care unit (HMU, ITU).
In this question, option four has a CURB-65 score of 1 and outpatient treatment can
therefore be considered. All the others are indications for inpatient treatment.
Clinical judgement must still however be used particularly if only one feature is present or if
there is co-morbidity such as age over 50 years or chronic cardiac, respiratory or renal
disease.

Work Smart
Time taken: 06:58

A 41-year-old female is brought into the Emergency department after taking an uncertain
quantity of paracetamol two hours previously and trying to hang herself.
She becomes agitated and insists that she wants to go home immediately. You judge that
she is at high risk of suicide.
Which of the following is the most appropriate course of action for this patient?

(Please select 1 option)


Ask her to sign a 'discharge against medical advice' form and let her go
Call the duty psychiatrist, but let the patient go if she insists and the duty psychiatrist does not arrive
in time to see her
Call the hospital security services, restrain her and sedate her
Detain her under section 5(2) of the Mental Health Act
Detain the patient under common law, seeking the help of hospital security services

Correct

This patient has taken an unknown quantity of paracetamol tablets, and continues to have
suicidal ideation. In situations such as these you need to assess whether the patient has
capacity.
Taking a paracetamol overdose in itself does not mean the patient has no capacity, but often
capacity is lacking. The chance of suffering severe consequences if a paracetamol overdose
goes untreated is high. Often, therefore these patients do not possess the level of capacity
required for a decision of this importance. Family should be involved if possible.
In cases such as this the patient, if found to lack capacity, should be held under common law
until the appropriate time to take a blood sample (four hours after the overdose).
Psychiatric team advice can be sought, but they need the patient to be 'medically fit' to be
able to do a full assessment. In general this means they must have completed all treatment
necessary (for example,. NAC infusion) prior to mental health assessment.
Allowing self-discharge in this situation would be inappropriate.
Sedation is inappropriate in this situation.
Section 5(2) of the Mental Health Act is used for patients who are already admitted to the
hospital who have a mental illness, to allow compulsory detention for up to 72 hours. Formal
assessment under the Mental Health Act should be undertaken as soon as possible. It is not
used in the Emergency department.

Work Smart
Time taken: 07:33

A 17-year-old male presents to the Emergency Department after an overdose of alcohol and
paracetamol.
He complained of abdominal discomfort and an intravenous infusion of N-Acetylcysteine was
commenced. Fifteen minutes later he developed breathlessness, reported feeling flushed
and developed a tachycardia.

What is the cause of this reaction?


(Please select 1 option)
A disulfiram-like (antabuse) reaction has occurred
The patient has developed pulmonary oedema
The patient has had a panic attack
The patient has received an overdose of N-Acetylcysteine
The patient has received N-Acetylcysteine previously

Correct

This patient is having an acute hypersensitivity reaction the most common, dose
independent adverse drug reaction. It is caused by previous exposure and being sensitised
to the drug. The initial exposure induces the production of antibodies of Ig E class,
subsequent exposure induces an immunological reaction - anaphylaxis. Some drugs can
produce an similar pseudoallergic reaction on first exposure.
The majoirity of dose-related adverse reactions occur within the first hour of the initial
infusion of acetylcysteine. The MHRA now recommends extending the time of the initial
infusion from 15 minutes to 60 minutes in order to reduce the incidence of adverse
reactions. Even if a patient has a history of a previous reaction to intravenous acetylcisteine,
the benefits outweigh the risks and patient should receive treatment. Any 'hypersensitivitylike' reactions are more likely to be anaphylactoid in nature (i.e. not immunologically
mediated) and therefore may not occur on repeated exposure.

Work Smart
Time taken: 07:40

Theme:Emergency medicine
A

Boerhaave's syndrome

Cardiac tamponade

Diaphramatic rupture

Dissecting thoracic aorta

Flail chest

Mallory-Weiss syndrome

Ruptured thoracic aorta

Tension pneumothorax

Traumatic haemothorax

Select the most likely diagnosis in the following cases.


A 25-year-old man who was involved in a high-speed motorbike crash is brought to the
emergency department with respiratory distress and left-sided chest pain. On examination, he
has distended neck veins and there is decreased air-entry on the left side of the chest. His blood
pressure is 100/72 mmHg, pulse rate 110/min and respiratory rate 20/min.

Incorrect - The correct answer is Tension pneumothorax

Tension pneumothorax is a life-threatening surgical emergency, since failure to relieve the


tension may result in a cardio-respiratory arrest. It usually occurs following penetrating or
blunt injuries to the chest, and frequently following major traumas.
In tension pneumothorax, the air is drawn into the pleural space with each inspiration, but
has no route to escape; thus acting as a one-way valve.
Patients present with
respiratory distress
tachycardia
hypotension
distended neck veins
decreased air-entry in the affected lung
deviation of trachea and mediastinum to the opposite side.

However, not all these signs and symptoms are always be present.

A 56-year-old man is brought by ambulance to the Emergency Department after being found
lying in the street. He complains of severe pain over the retrosternal and epigastric region
following a bout of heavy drinking. He also gives a history of vomiting blood before the onset of
pain.
On examination he is hypotensive with a tachycardia. A chest x ray shows gas in the
mediastinum and subcutaneous tissues.

Correct

The classical history of Boerhaave's syndrome is of severe vomiting and retching followed
by extremely severe retrosternal and upper abdominal pain. Shock develops rapidly.
There is a history of alcoholism or heavy drinking in 40% of patients. The site of rupture is
usually in the left posterolateral distal oesophagus and is several centimetres long.
Subcutaneous emphysema (crepitus) is only present in 27% of patients and is a relatively
late sign.
An initial chest x ray will show mediastinal or free peritoneal gas. After hours or days, pleural
effusion(s), often with a pneumothorax, and a widened mediastinum develops. The
diagnosis is confirmed with a CT scan followed by a gastrografin swallow to assess the
extent of the oesophageal leak.
The main treatment is surgery, which should be within 24 hours. Mortality is 20-50% and is
increased with delay in treatment. The oesophagus is repaired or resected and the
mediastinum drained. Occasionally contained leaks may be managed conservatively.
Endoscopic covered stents have been used. Surgery is the only effective option when there
is extensive mediastinal contamination or delay in diagnosis.
Mallory-Weiss syndrome is the cause of bleeding in 5% of patients with upper
gastrointestinal haemorrhage. Longitudinal mucosal lacerations in the distal oesophagus
and proximal stomach cause bleeding from submucosal arteries. Most tears are single. The
condition was originally described in 1929, related to vomiting in alcoholic patients.
Other associations include
coughing
epileptic convulsions
closed chest massage
blunt abdominal injury

hiccups under anaesthesia.

Hiatus hernia appears to be a predisposing factor (40-100%). Some patients have epigastric
or back pain. The blood loss is usually small and self-limiting.
Transfusions may be needed and endoscopic haemostatic treatment may be required.
Rarely, with protracted vomiting, perforation may occur.

A 37-year-old man is brought into to the Emergency Department with penetrating injury to the left
side of his chest wall following a road traffic accident. He complains of severe left-sided chest
pain, and on examination, his jugular venous pressure (JVP) is raised and the heart sounds are
muffled. His blood pressure is 98/74 mmHg and his chest x ray reveals a globular heart.

Incorrect - The correct answer is Cardiac tamponade

Cardiac tamponade may occur following


penetrating or blunt injuries to the chest wall and/or heart
lung or breast carcinomas
pericarditis
myocardial infarction.

The classical signs of cardiac tamponade include a rising JVP, falling BP and muffled heart
sounds (Beck's triad).
The other recognised features include a rising JVP with inspiration (Kussmaul's sign),
tachycardia and hypotension.
Chest x ray reveals a globular heart and the left heart border may be convex or straight with
the right cardiophrenic angle reduced to less than 90.

Work Smart
Time taken: 07:53

Ventricular rather than supraventricular tachycardia is suggested by which of the following?

(Please select 1 option)


A pre-excitation pattern on the ECG after reversion to sinus rhythm
A variable intensity of the first heart sound

This is the correct answer

An rSR' pattern on the ECG in lead V1


Irregular QRS complexes on the ECG
Slowing of the heart with carotid sinus massage

Incorrect answer selected

Ventricular tachycardia produces AV dissociation which gives rise to diagnostic clinical signs
(variable intensity of the first heart sound, cannon waves in JVP) and ECG features (capture
beats and fusion beats).
Vagal manoeuvres such as carotid sinus massage slow down AV nodal conduction - SVTs
that rely on the AV node for propagation will be affected while VT will not.
Pre-excitation suggests Wolff-Parkinson-White syndrome (WPW), which can cause a broad
complex tachycardia (anterograde conduction via bundle of Kent) although a narrow
complex tachycardia is more common.
When right bundle branch block (RBBB) occurs in VT, it is usually an Rsr' pattern, while an
rsR' pattern suggests SVT with rate-dependent bundle branch block. VT is regular - consider
AF with aberrant conduction if irregular.

Work Smart
Time taken: 08:01

Which of the following do not tend to make the symptoms of myasthenia gravis (MG) worse?
(Please select 1 option)
Aminoglycosides
Lithium
Prednisolone

This is the correct answer

Pyridostigmine
Quinidine

Incorrect answer selected

Pyridostigmine is used in the treatment of myasthenia, but in the initial stages of treatment, it
may make the symptoms worse.
Prednisolone may be used for the treatment of MG if anticholinesterase treatment is failing.

The main drugs to be regarded with caution are diuretic agents, which may lead to
hypokalaemia, thus precipitating a myasthenic crisis.

Work Smart
Time taken: 08:07

Which of the following investigations would contribute the most to acute risk stratification in a
patient with unstable angina?
(Please select 1 option)
Coronary angiography

Echocardiography

Electrocardiography

Myocardial perfusion scanning

Troponin testing

Correct

The adverse prognostic factors in unstable angina are


Labile ECG changes
Persistent angina in spite of maximal medical therapy
Clinical evidence of heart failure.

However, the most important adverse prognostic factor is troponin elevation. These are the
patients who should be referred for urgent coronary revascularisation

Work Smart
Time taken: 08:12

A 40-year-old male presents to the Emergency department.


Routine admission bloods reveal a triglyceride level of 20 mmol/l (0.45-1.69).
Which of the following conditions can be precipitated by hypertriglyceridaemia?
(Please select 1 option)
Acute pancreatitis

This is the correct answer

Hypothyroidism

Nephrotic syndrome

Thiazide therapy

von Gierke's

Incorrect answer selected

Hypertriglyceridaemia can cause acute pancreatitis.


The most common causes of hypertriglyceridaemia are as follows:
Obesity
Alcohol intake, particularly if excessive
Hypothyroidism
Insulin resistance and poorly controlled diabetes mellitus
Pregnancy.

Familial causes also exist including familial hypertriglyceridaemia and familial combined
hyperlipidaemia (high trigs and high cholesterol).
In patients with persistent high triglycerides treatment is advisable to reduce the risk of
pancreatitis.
High triglycerides are also thought to be related to cardiovascular events.

Work Smart

Time taken: 08:15

A 51-year-old politician experiences palpitations, and when he receives an ECG it


demonstrates a regular narrow complex tachycardia of 180 bpm.
Which of the following is not a recognised problem with the use of intravenous adenosine in
this patient?
(Please select 1 option)
Bronchospasm if the patient is asthmatic
Possible worsening of re-entry tachycardia if this patient has Wolff-Parkinson-White syndrome
Production of chest pain even if this patient has normal coronary arteries
The need for dose reduction if the patient is on disopyramide
The need for dose reduction if the patient is on theophylline

Correct

The action of adenosine is blocked by theophylline, and enhanced by disopyramide. The


patient should be warned about chest tightness and dizziness, and a defibrillator should be
at hand in case the tachycardia accelerates, as may happen with the Wolff-Parkinson-White
syndrome.
Adenosine can produce profound bronchospasm and should be avoided in asthmatics.

Work Smart
Time taken: 08:19

A 49-year-old post-woman with a long-standing diagnosis of acute intermittent porphyria


(AIP) is referred by her general practitioner for advice on drug therapy.
She is a longstanding poorly-controlled hypertensive. The GP has provided two
electrocardiograms from consecutive weeks both of which demonstrate atrial fibrillation.
She has not had an acute exacerbation of AIP for over seven years.
Which of the following drugs would be unsuitable for use in her treatment?
(Please select 1 option)
Aspirin

Atenolol

Bumetanide

Digoxin

Methyldopa

Correct

AIP is often associated with hypertension.


Centrally acting drugs, such as the following, are contraindicated:
methyldopa
clonidine
angiotensin-converting enzyme (ACE) inhibitors
calcium channel blockers, and
furosemide.

Among the diuretics, the following have been used safely:


amiloride
bumetanide
acetazolamide
cyclopenthiazide, and
triamterene.

Digoxin, beta-blockers, heparin, and warfarin are also thought to be safe

Work Smart
Time taken: 08:24

Which of the following is associated with a prolonged QT interval?


(Please select 1 option)
Digoxin

Hyperkalaemia

Hyperthermia

Hypocalcaemia

Hypokalaemia

This is the correct answer

Incorrect answer selected

Causes of QT prolongation include


Congenital long-QT syndromes
Drugs: Class I and Class III anti-arrhythmic drugs
Tricyclic antidepressants, phenothiazines, non-sedating antihistamines
Hypocalcaemia
Hypothermia
Severe bradycardia.

Causes of a short QT interval include


Hypercalcaemia
Administration of magnesium
Digitalis toxicity.

Potassium alterations do not commonly affect the QT interval.


Hypokalaemia produces flattening of the T wave and a pronounced U wave.

Work Smart

Time taken: 08:29

Which of the following is not a recognised effect of chronic cocaine abuse?


(Please select 1 option)
Hallucinations
Hypersomnia
Hyponatraemia

This is the correct answer

Severe anxiety and paranoid ideation


Sexual dysfunction in men

Incorrect answer selected

Chronic users of cocaine often feel that they perform better in many areas when intoxicated.
This misperception probably results from the overriding effects of euphoria and stimulation
caused by the drug.
Chronic cocaine use can result in erectile dysfunction, ejaculatory dysfunction, hypersomnia.
Severe anxiety and paranoid hallucinations increase with more frequent use.

Work Smart
Time taken: 08:35

Reversed splitting of the second heart sound is found in which one of the following?
(Please select 1 option)
Atrial septal defect
Left bundle branch block (LBBB)

This is the correct answer

Mild aortic stenosis (AS)


Right bundle branch block (RBBB)
Ventricular septal defect (VSD)

Incorrect answer selected

Reversed splitting of the second heart sound occurs with reversal of the normal A2, P2
pattern. Thus A2 may be delayed as with severe AS (not mild), and LBBB. P2 may be early
as with Wolff-Parkinson-White type B and persistent ductus arteriosus.
Atrial septal defects show wide fixed splitting, and RBBB has wide, but not fixed splitting.

Work Smart
Time taken: 08:41

Abdominal pain is a feature of all of the following except which?


(Please select 1 option)
Acute arsenic poisoning
Acute iron poisoning
Acute lead poisoning
Methanol toxicity
Typhoid fever

Correct

Lead toxicity acutely leads to colicky abdominal pains, fatigue, paralysis and
encephalopathy.
Arsenic chronically causes hyperkeratosis, tranverse nail bed lines (Mees' lines) and chronic
vasospasm causing blackfoot disease. Acute toxicity causes severe abdominal pain,
haemorrhagic gastroenteritis and hepatic necrosis. Capillary leaking can lead to pulmonary
oedema and shock. Later a peripheral neuropathy develops.
Methanol toxicity causes a metabolic acidosis, optic atrophy and peripheral neuropathy. After
a delay of 4-36 hours, nausea, vomiting and abdominal pain are seen.
Pain is not a dominant feature of typhoid fever. The most constant accompaniment of the
fever in typhoid is headache. Along with this go non-specific symptoms, to be expected of a
significant infection (malaise, lassitude, myalgia, arthralgia and anorexia).
In classical typhoid, constipation is a frequent early symptom though the majority of patients
will experience loose motions at some time. Bloody diarrhoea may be seen. Nausea and
vomiting are relatively infrequent in uncomplicated typhoid but are seen with abdominal
distention in more severe cases.
Other symptoms of early typhoid are cough, sore throat and a tendency to epistaxes.

Work Smart
Time taken: 08:48

A 39-year-old female escort attends her local GU clinic with a fever, malaise and arthralgia.
She has a widespread maculo-papular rash, but no other abnormalities.
Investigations show
HIV p24 antigen

negative

Cold agglutinin IgM antibody

1:32

FT-ABS

negative

VDRL

positive

CRP

137

The patient recovers but four weeks later presents to the emergency department with double
vision and difficulty walking. Deep tendon reflexes are absent.
Which is the most important test to arrange next?
(Please select 1 option)
Lumbar puncture
MRI head scan
Nerve conduction studies
Spirometry
Tensilon test

This is the correct answer


Incorrect answer selected

The most likely cause of the presenting illness is Mycoplasma pneumoniae infection.
This can cause fever, arthralgia, maculopapular rash and a raised CRP. It is also a cause of
false positive VDRL and positive cold agglutinins.
Neurological syndromes post infection include meningoencephalitis, transverse myelitis and
Guillain-Barre syndrome.
MRI, lumbar puncture and nerve conduction studies are all indicated but the most important
investigation is measurement of vital capacity to monitor any evolving respiratory paralysis.

Work Smart
Time taken: 08:53

A 64-year-old woman presented 10 hours after ingestion of 12 g of quinine sulphate.

Which of the following is the most common characteristic clinical feature in this situation?
(Please select 1 option)
Blindness

This is the correct answer

Bradycardia
Hyperacusis
Hyperglycaemia
Hypotension

Incorrect answer selected

The major toxic effects of quinine are on the nervous system, in particular the optic and
auditory nerve. It is also particularly toxic on retinal photoreceptor cells, and causes
vasoconstriction and spasm of the retinal artery. Blurred vision can proceed to complete
blindness, which is characteristic of overdose. Initial signs are narrowing of the retinal
arterioles on fundoscopy. Later there is retinal oedema, and the pupils become dilated and
unresponsive to light.
Effects on the auditory nerve may cause tinnitus and deafness, but not hyperacusis.
Initially quinine causes generalised stimulation of the central nervous system, leading to
tachycardia, fever, delirium and tachypnoea. In severe overdose it can then cause
myocardial depression, peripheral vasodiltation and widened QRS with prolonged QTc and
risk of VT, torsade and VF.
In addition, renal failure, haemolytic anaemia and gastrointestinal disturbance can develop.
At very high plasma levels, sinoatrial block and high-degree atrioventricular block may be
seen. Hypotension may occur secondary to quinine's alpha-blocking effect, but is not as
characteristic as blindness.
Additional features include nausea, vomiting, headache, seizures, fatigue, tremor and
ataxia. Hyperglycaemia is not characteristic.
Management is with activated charcoal and gastric lavage, if presentation is early enough.
Bradycardia can be treated with atropine or pacing. Bicarbonate is considered for a
prolonged QRS (>120ms), aiming for a pH of 7.45-5. Haemodialysis can also be used to
clear the quinine. There is no specific treatment for visual disturbance.

Work Smart
Time taken: 08:58

A 16-year-old girl presented with fever, headache and photophobia.


Cerebrospinal fluid examination revealed:

Opening pressure

260 mm H2O

(50-180)

Total protein

0.8 g/L

(0.15-0.45)

Glucose

4.2 mmol/L

(3.3-4.4)

White cell count

60 per ml

(<5)

Lymphocytes

90%

(60-70)

Plasma glucose

6.4 mmol/L

(3.0-6.0)

What is the most likely diagnosis?


(Please select 1 option)
Bacterial meningitis

Cryptococcal meningitis

Tuberculosis meningitis

Viral encephalitis

Viral meningitis

Correct

Normal cerbrospinal fluid (CSF) glucose together with


a CSF lymphocytosis
an increased opening pressure, and
a raised CSF protein

are typical of a viral meningitis, which would be high on the list of differentials in patients of
this age group (together with bacterial meningitis).

Work Smart

Time taken: 09:03

A 72-year-old man presented following three episodes of transient loss of consciousness not
associated with chest pain. There was a previous history of an anterior myocardial infarction.
On examination his blood pressure was 140/80 mmHg and the apex beat was diffuse in
character and displaced to the left. There were no neurological signs.
The ECG showed sinus rhythm with occasional ventricular extrasystoles, deep anterior Q
waves and ST segment elevation in leads V2 - V6, without reciprocal depression.
Which one of the following would be the most appropriate initial course of action?
(Please select 1 option)
Administer tissue plasminogen activator
Arrange an electroencephalogram
Arrange immediate computerised tomography (CT) brain scan
Observe in the coronary care unit
Proceed to coronary arteriography

This is the correct answer


Incorrect answer selected

The history suggests that this man has persistent ST elevation in the anterior leads, with a
previous history of anterior myocardial infarction due to left ventricular (LV) aneurysm. The
blackouts are cardiac syncope due to rhythm disturbance. An ECHO would quickly support
the diagnosis but, because of the risk of sudden death, observation on CCU is required.
The loss of consciousness is likely to be due to recurrent arrhythmic episodes or
vertebrobasilar transient ischaemic attacks (TIAs), as a result of embolisation of an LV
thrombus.
The most important investigation for this patient whom you suspect has arrhythmic episodes
would be telemetry/24 hour monitoring, and hence observation on the coronary care unit
(CCU) is appropriate.

Work Smart
Time taken: 09:06

A 16-year-old girl presents with an acute exacerbation of asthma.

On examination her respiratory rate was 30 per minute, her heart rate was 120 beats per
minute and a peak expiratory flow rate (PEFR) was 30% of the predicted value.
Her blood gas analysis on air shows:
PaO2

9 kPa

(11.3-12.6)

PaCO2

3.5 kPa

(4.7-6.0)

After the administration of oxygen and corticosteroids what is the most appropriate next step
in management?
(Please select 1 option)
Intravenous aminophylline
Intravenous salbutamol
Ipratropium bromide via oxygen-driven nebuliser
Salbutamol via oxygen-driven nebuliser
Salmeterol via breath-actuated inhaler

This is the correct answer


Incorrect answer selected

This patient fits the criteria for acute severe asthma. In such cases 2-agonists should be
administered as soon as possible, preferably nebulised driven by oxygen. Repeat doses
should be given at 15-30 minute intervals, or continuous nebulisation can be used where
there is inadequate response to bolus therapy. Nebulised ipratropium bromide should be
added for patients with acute severe or life threatening asthma, or those with a poor initial
response. It's addition produces significantly greater bronchodilation than a 2-agonist
alone.
Oxygen should be given to maintain saturations at 94-98%. Patients with saturations less
than 92% on air should have an ABG to exclude hypercapnia. However, starting treatment
should not be delayed to do the ABG. Initially high-flow oxygen is used, and then weaned to
maintain adequate saturations. Unless you suspect COPD there isn't a need to be cautious
with oxygen therapy.
Steroids reduce mortality, relapses, subsequent hospital admission and requirement for 2agonists1. The earlier they are given in the attack, the better the outcome. A dose of 4050mg should therefore be given once oxygen and nebuliser therapy has been established.
This should be continued for 5 days, or until recovery, and can then be stopped abruptly
unless the patient has taken long-term oral corticosteroids.
Failure to respond to the above treatment steps may warrant the use of intravenous
magnesium sulphate and aminophylline. However, these should not be used without
discussion with your senior colleagues.
Intensive care is indicated for patients with severe acute or life threatening asthma who are
failing to respond to therapy. Consider it in patients with deteriorating peak flow, persisting or
worsening hypoxia, hypercapnia, acidosis, exhaustion or altered conscious state. All patients
who are transferred to an intensive care unit should be accompanied by a doctor who can
intubate if necessary.

As an aside, chest radiographs are not indicated unless you suspect pneumothorax or
consolidation, or there is life-threatening asthma, a failure to respond to treatment or a need
for ventilation.

Work Smart
Time taken: 09:10

A 17-month-old boy has been completely well and playing with his toys. Mother hears him
coughing and choking and rushes next door.
She finds him gagging and retching, drooling and red in the face.
He was born at 37+3/40 weighing 3.7 kg and there were no neonatal problems.
On examination his temperature is 36.9C, respiratory rate 30/min, heart rate 130/min. He
looks slightly cyanosed with an O2 saturation of 85% in air. He has marked stridor and
moderate recession.
What is the most likely diagnosis?
(Please select 1 option)
Anaphylaxis
Croup
Foreign body aspiration

This is the correct answer

Peritonsillar abscess
Retropharyngeal abscess

Incorrect answer selected

The history suggests aspiration of a foreign body.


Small objects, such as toys, are the commonest offenders. They can give rise to acute
obstruction, or there may be a latent period when symptoms settle before inflammation
around the object gives rise to symptoms.
Assessment requires a chest x ray, which will locate radioopaque objects, and bronchoscopy
to remove the object.

Work Smart

Time taken: 09:15

You are called to the delivery of a 41+4 gestation infant who is being delivered by
emergency caesarian section. Mother is a healthy Caucasian of 27 years, who smokes 15/d.
There have been concerns about fetal growth on serial ultrasound scans.
Labour commenced 18 hours ago, and mother has been in Stage 2 for the past 90 minutes.
There have been three bradycardias with delayed recovery, and the liquor is thickly
meconium stained.
The baby is given to you. She is floppy, pale and covered in thick meconium.
What is the most likely diagnosis?
(Please select 1 option)
Group B Strep septicaemia
Holoprosencephaly
Hypoxic ischaemic encephalopathy
Meconium aspiration

This is the correct answer

Surfactant deficient lung disease

Incorrect answer selected

This child is growth retarded and has had significant fetal distress (birth asphyxia). A cord
gas may help judge the severity of the perinatal insult.
She should be quickly wiped and wrapped in a warm towel, then placed head down on the
resuscitation trolley.
If vigorous no further action is required. If flat, her oropharynx and cords should be inspected
and suction applied to the trachea if there is suspicion that there is some meconium between
them.
She should then be intubated and ventilated. Depending on her response she may require
external cardiac massaage (ECM) or a bolus of fluid, e.g. given via the umbilical vein
catheter (UVC).

Work Smart
Time taken: 09:19

A 17-year-old female presents with a severe headache of sudden onset associated with
nausea and photophobia.

On examination there was evidence of meningism. She had a previous history of migraine
without aura and was taking the oral contraceptive pill.
Which of the following is the most appropriate initial investigation?
(Please select 1 option)
Cervical spine x ray
CT scan of head

This is the correct answer

Lumbar puncture
MRI scan of head
Skull x ray

Incorrect answer selected

This presentation suggests the diagnosis of subarachnoid haemorrhage.


The first line investigation is a head CT scan looking for evidence of haemorrhage.
If this is not demonstrated it should be followed by a lumbar puncture as subarachnoid blood
may not be evident on up to 10% of cases on CT scan.

Work Smart
Time taken: 09:26
PLAB 2003

Theme:Trauma management
A

CT scan head

Endotracheal intubation

Intercostal drain

Laparotomy

Large bore IV access and fluid resuscitation

Log roll and rectal examination

Needle cricothyroidotomy

Needle decompression

Pressure dressing

Thoracotomy

Select the most appropriate next management step from the given list for the following
patients:
A 25-year-old male was brought into the Emergency department following a motorbike accident.
He was found unconscious lying prone about 25 meters from the bike. He had been intubated at
the site. His neck was protected with a collar. He was being mechanically ventilated with 100%
oxygen in the emergency department. He had a thready pulse of 100/min, blood pressure of
70/50 mmHg, SaO2 of 90%. Trachea is central but air entry is decreased on the left side with a
tympanic note on percussion.

Incorrect - The correct answer is Needle decompression

A 20-year-old female horse rider was brought into the emergency department on a spinal board
having fallen of her horse. She was complaining bitterly about being restrained on the spinal
board because her back was hurting. On examination, she had a pulse of 120/min, blood
pressure of 84/30 mmHg, normal chest examination. Abdominal examination showed a bruise
and tenderness on her left hypochondrium and lumbar regions. She had a decreased sensation
below her knees and she could not move her toes.

Incorrect - The correct answer is Large bore IV access and fluid resuscitation

A 55-year-old male was admitted after having a high speed accident in his car. He was found
with his head on the steering wheel. His airway was patent, neck protected with appropriate
collar and intravenous (IV) access secured. His primary survey revealed severe facial injuries,
Glasgow Coma Scale (GCS)of 13/15 and probable pelvic and bilateral femoral fractures. During
secondary survey, his respiration was noted to be laboured with gurgling sounds and GCS
suddenly dropped to 6/15.

Incorrect - The correct answer is Endotracheal intubation

In situations of trauma, it will help to remember ABCD.


In all circumstances management of airway comes before breathing; breathing before
circulation; circulation before dysfunction/disability. The answers to above scenarios are
based on these principles.
Clinically this young male in a road traffic accident has a pneumothorax on the left side. The
immediate treatment of this is needle thoracotomy/decompression buying time for a more
definitive insertion of an intercostal drain.
The female involved in a horse riding accident probably has a spinal injury and may also
have a splenic haemorrhage but before proceeding further she needs venous access and
fluid resuscitation.
The 55-year-old male has developed an airway problem and so needs definitive airway
management prior to urgent CT head scan.

Work Smart
Time taken: 09:34

Theme:Treatment of burns
A

Anaesthetise and intubate

Deroof blisters

Dressing

Emollient cream

Escharotomy

Ice packs

Intravenous fluids

Intravenous opiates

Irrigate with cold water

Referral to the specialised burns unit

Select the most appropriate immediate treatment for the following patients:
A 52-year-old male electrician sustained full thickness burns to all the fingers of his dominant
hand after grabbing a hot welding rod.

Incorrect - The correct answer is Referral to the specialised burns unit

This man has burnt his fingers and needs specialist care to prevent complications and
complete rehabilitation.

A 49-year-old female is admitted with 40% burns sustained in a camping accident with a
kerosene lamp.

Incorrect - The correct answer is Intravenous fluids

The woman has 40% burns. In general burns greater than 15% require intravenous fluids.

A 33-year-old male is admitted to the Emergency department after a fire accident with petrol in
his garage. He is conscious, breathless but vital signs are fine; however he has singed his
nostrils and has pain in his throat.

Correct

The issue in the man involved in a garage fire is the potential major airway damage. His
symptoms and signs indicate upper airway and probable lung injury. Although there is no
immediate airway problem, this patient may benefit from anaesthesia and intubation to
protect his airway and reduce the risk of pulmonary complications.

Burns management is dictated by the extent and degree of burns. The extent of burns in an
adult is calculated by the 'rule of nines' and the degree by the depth of the burn - partial
thickness or full thickness.
Inhalation injury greatly increases the mortality of burn patients. Indicators of such injury are:
Burns sustained in a closed space
Facial or oropharyngeal burns
Singed nasal hair, and
Carbonaceous sputum.

Such patients may benefit from early prophylactic intubation and ventilation.

Work Smart
Time taken: 09:41

Theme:Acute abdominal conditions in adults


A

Appendicitis

Carcinoma of the ileo-caecal region

Carcinoma of the rectum

Crohns disease

Ischaemic colitis

Mittelschmerz

Pelvic inflammatory disease

Ruptured ectopic pregnancy

Torsion of the ovary

Ulcerative colitis

Select the most probable diagnosis explaining the presentation of the following cases:

A 25-year-old lawyer presents to the surgical out-patient clinic with an 8-week history of rightsided lower abdominal pain, loss of appetite and diarrhea. She opens her bowels 10-12
times/day and the stools are mixed with mucus. She has recently noticed some abscesses in her
perianal region. Her blood tests reveal a microcytic, hypochromic anaemia and raised
inflammatory markers.

Incorrect - The correct answer is Crohns disease


Crohns disease is an idiopathic, chronic, transmural inflammatory disorder of the bowel that can
affect any part of the gatro-intestinal tract from the mouth to the anal canal. Most cases, however,
involve the small bowel, particularly the terminal ileum. The disease is associated with
ulcers/abscesses/sinuses/fistulas in the anal/perianal region. The patients may present with
abdominal pain, intermittent fevers, secondary anaemia, loss of appetite, loss of weight, diarrhoea
and the passage of blood and/or mucus per rectum. Chronic Crohns disease is associated with
stricture in the bowels, sinus and fistula formation

A 21-year-old woman is brought to the Accident and Emergency department with severe, rightsided lower abdominal pain and mild vaginal bleed. She is pale, and her pulse rate is 112/min
and blood pressure is 94/66 mmHg. Her last menstrual period was 6-weeks ago.

Incorrect - The correct answer is Ruptured ectopic pregnancy


Ectopic pregnancy occurs in less than 1% of pregnancies. The early signs of ectopic pregnancy
includes missed periods, minor spotting at the time of the missed periods and vaginal bleeding. There
may be other associated signs of pregnancy such as mastalgia, morning sickness and increased
urinary frequency. In ruptured ectopic pregnancy the abdominal pain is initially crampy, but
subsequently becomes a more continuous and generalized over the lower abdominal pain. Severe
pain is caused by the tubal rupture and also due to discharge of large quantity of blood into the
peritoneal cavity. The degree of shock depends on the amount of blood loss.

An 81-year-old woman, who is in known atrial fibrillation, is brought to the Accident and
Emergency department with an 8-hour history of severe, colicky pain in the peri-umbilical region,
vomiting and per rectal bleeding. Her blood pressure is 104/72 mmHg and her pulse rate is
94/min. Her bowel sounds are absent. Arterial blood gas analysis reveals elevated lactate levels
and metabolic acidosis.

Incorrect - The correct answer is Ischaemic colitis


Ischaemic colitis (also known as Mesenteric Infarction) is caused by structural changes to the colon
as a result of deprivation of blood, such as embolism, commonly affecting the superior mesenteric
artery. The occlusion may be due to a thrombus or an embolus and is seen in elderly patients who are
in atrial fibrillation. The signs and symptoms of acute mesenteric infarction include sudden onset of
severe lower abdominal pain, vomiting, passage of blood per rectum and shock. The inflammatory

markers may be elevated and the arterial blood gas analysis may reveal an elevated lactate level and
metabolic acidosis.

Work Smart
Time taken: 09:47
PLAB September 2010

A 50-year-old man has an appointment for a routine blood pressure check. He is sitting in
the waiting room when he becomes very breathless with stridor.
He had mentioned to another person in the room that he had been stung by an insect on his
way to the clinic. You diagnose an anaphylactic reaction to the sting.
What dose and route of administration would you use for adrenaline in this situation?
(Please select 1 option)
Intramuscular 1:1000 (500 micrograms)

This is the correct answer

Intramuscular 1:10000 (50 micrograms)


Intravenous 1:1000 (1 mg)
Subcutaneous 1:1000 (1 mg)
Subcutaneous 1:10000 (50 micrograms)

Incorrect answer selected

The recommended route of injection is intramuscular. The subcutaneous route can be used
but is not as effective. Occasionally intravenous adrenaline 1:10000 is used but only under
specialist supervision.
Further information on emergency treatment of anaphylactic reactions and guidelines for
health care providers can be found on the Resuscitation Council (UK) website.

Work Smart
Time taken: 09:51
PLAB September 2010

A 58-year-old man who has a history of hypertension and type 2 diabetes presents to the
Emergency department complaining of central chest pain which is going down his left arm.
His medication includes ramipril, metformin, atorvastatin and gliclazide.

On examination his BP is 129/72 mmHg, and his pulse is 81. He has bibasal crackles on
auscultation of his chest.
Investigations reveal:
Haemoglobin

13.8 g/dl

(13.0-18.0)

White cell count

8.9 109/l

(4-11)

Platelet

197 109/l

(150-400)

Sodium

141 mmol/l

(135-146)

Potassium

4.1 mmol/l

(3.5-5)

Creatinine

123 mol/l

(79-118)

Glucose

12.3 mmol/l

(<7.0)

ECG

Anterolateral ST depression

He is given sublingual GTN.


Which of the following is the next most appropriate therapy?
(Please select 1 option)
Aspirin 300 mg and low molecular weight heparin
Aspirin 300 mg, clopidogrel 300 mg and low molecular weight heparin

This is the correct answer

Beta blockade
Clopidogrel 300 mg
IV GTN infusion

Incorrect answer selected

The answer is aspirin, clopidogrel and low molecular weight heparin.


This patient is high risk given his history of type 2 diabetes mellitus, and as such should be
loaded with both aspirin and clopidogrel.
Further chest pain, or failure of his ECG signs to resolve may drive further intervention
including progression to angiography.
If this patient does not progress to angiogram then screening for ischaemia should be
considered prior to discharge.

Work Smart

Time taken: 10:29


PLAB September 2010

A 23-year-old woman is admitted with pain on passing urine and fever. She says she has
been unwell for the past three days and has now developed left loin pain. Only medication of
note is the combined oral contraceptive pill.
On examination she is pyrexial 38.8C and has a BP of 105/60 mmHg, her pulse is 94 and
regular. She has left renal angle and supra-pubic tenderness.
Investigation shows:
Haemoglobin

11.4 g/dl

(11.5-16.5)

White cells

14.9 109/l

(4-11)

Platelet

193 109/l

(150-400)

Sodium

140 mmol/l

(135-146)

Potassium

4.1 mmol/l

(3.5-5)

Creatinine

132 mol/l

(79-118)

Urine blood, protein ++

Which of the following is the most appropriate course of action?


(Please select 1 option)
Advise her to take cranberry juice and discharge
Arrange an outpatient CT of the abdomen
IV antibiotics
Start trimethoprim and discharge
Ultrasound abdomen

Correct

The answer is ultrasound abdomen.


The left renal angle pain raises the possibility of pyelonephritis and significant ureteric
obstruction therefore needs to be excluded.
The best way to do this is by ultrasound, looking for signs of obstruction. If ureteric dilation is
seen then an opinion from a urologist should be obtained.
Given her raised white count, elevated creatinine and significant hypotension, IV antibiotics
with a broad spectrum cephalosporin should be considered.

Work Smart
Time taken: 10:33

A 4-year-old girl attends the Emergency department after informing her mother that she has
just put a small bead into her ear.
You examine both ears and confirm the presence of a plastic bead deep in the external
canal of her right ear.
Which of the following is the most appropriate course of action?
(Please select 1 option)
Attempt to remove the bead under direct vision
Prescribe topical antibiotic drops
Reassure the child and her mother that the bead will eventually fall out on its own
Refer the patient for emergency admission and retrieval of the foreign body under general
anaesthetic
Refer the patient for review in ENT clinic

Correct

Ear foreign bodies are common in all age groups and rarely constitute a real emergency.
Uncomplicated foreign bodies in the external canal can be referred to ENT and seen in the
next available clinic. Microscopic removal by skilled ENT staff, using appropriate equipment,
is usually possible.
If this fails, patients will require a general anaesthetic to remove the foreign body from their
ear. Removing foreign bodies in the Emergency department can be difficult with limited
equipment. Failed attempts can cause upset that thwarts subsequent attempts at retrieval.
If there is evidence of infection or trauma to the ear (blood visible or an abnormal tympanic
membrane) patients should be referred as an emergency.
Antibiotic prescription is only necessary if there is evidence of associated infection. Foreign
bodies should not be left in the ear because of the potential for causing infection.
Removal also permits a thorough examination of the ear to ensure that there are no
associated injuries, such as a tympanic membrane perforation.

Work Smart

Time taken: 10:41

A 28-year-old male attends the Emergency department following an alleged assault.


He reports sustaining a blow to his nose that caused a small nosebleed and considerable
swelling and deformity. He complains of difficulty breathing through his left nostril.
Your examination confirms that there is no active bleeding from the nose. There is some
deviation of the nasal bones to the left and no septal haematoma.
Which of the following is the most appropriate cause of action?
(Please select 1 option)
Arrange an ENT follow up appointment for within the next one week

This is the correct answer

Pack the patient's nose

Reassure the patient that his symptoms will resolve as the swelling subsides and needs no further
action

Refer the patient for emergency admission

Request x rays of the nasal bones

Incorrect answer selected

Isolated nasal injuries are a common presentation to the Emergency department. Patients
rarely require emergency admission.
There are three major exceptions to this, patients with
A septal haematoma
A compound nasal fracture and
Associated epistaxis

will all require emergency admission.


x Rays of the nasal bones are complex and unnecessary. Diagnosis of a nasal fracture is
entirely clinical.
Uncomplicated patients are best reviewed after five days in the ENT clinic, when associated
swelling has subsided and it can be determined whether manipulation of the fracture is
appropriate.

Traumatic epistaxis can be serious and may require packing if there is active bleeding.

Work Smart
Time taken: 10:47

A 76-year-old lady attends the Emergency department after knocking her shin on some
furniture at home. She takes prednisolone for polymyalgia rheumatica.
You examine her leg and find a pre-tibial laceration with a large skin flap.
What is the best way to manage this?
(Please select 1 option)
Clean the laceration and apply a non adherent dressing
Clean the laceration only
Steristrip the laceration

This is the correct answer

Suture the laceration with a 4/0 non-absorbable suture


Suture the laceration with an absorbable suture

Incorrect answer selected

In young patients with good skin, pre-tibial lacerations may be sutured, usually with nonabsorbable sutures that are removed after seven to 10 days.
In elderly patients with thin skin, or those on warfarin or steroids the skin is frequently too
fragile to suture. For these patients the wound should be thoroughly cleaned and
meticulously streristripped to best aid skin healing. A non-adherent dressing and a light
bandage can be applied and the patient should be encouraged to elevate the leg.
Patients should be reviewed after a week.
The laceration may take many months to heal and some require skin grafting procedures.

Work Smart
Time taken: 10:53

A 45-year-old carpenter attends the Emergency department after cutting his left index finger
with a saw.

On examination there is a 2 cm incised wound on the radial aspect of the index finger. The
neurovascular supply is intact and there is no evidence of injury to other structures.
Which is the appropriate local anaesthetic to use to explore the wound and suture the
laceration?
(Please select 1 option)
Lidocaine 1% - 20 ml

This is the correct answer

Lidocaine 1% - 250 ml
Lidocaine 1% with adrenaline - 20 ml
Lidocaine 1% with adrenaline - 250 ml
Topical ethyl chloride

Incorrect answer selected

For routine use 1% lidocaine is the most frequently used anaesthetic for cleaning, exploring
and suturing wounds in the Emergency department.
Adrenaline should not be used in sites supplied by end arteries, such as fingers and toes.
The maximum dose of plain lidocaine in a healthy adult is 3 mg/kg or 200 mg (20 ml of 1%).
Remember that 1% lidocaine = 10 mg/ml.
If lidocaine with adrenaline is used the maximum dose is 7 mg/kg or 500 mg (50 ml of 1%).
The duration of action is 30-60 minutes for plain lidocaine and approximately 90 minutes for
lidocaine with adrenaline.
Ethyl chloride is a topical anaesthetic that can be sprayed onto the skin and causes rapid
cooling. It is very short acting (lasts less than 60 seconds) and would provide inadequate
analgesia in this case.

Work Smart
Time taken: 10:57

A 58-year-old dairy farmer attends the Emergency department following a fall at work. He
slipped in the cowshed cutting his leg on some metal fencing.
On examination he has a severely contaminated deep wound on the lateral aspect of his
thigh.
After completing the examination and obtaining an x ray to exclude a foreign body you would
like to clean the wound.

Which cleaning agent is the best to use?


(Please select 1 option)
Hydrogen peroxide

Industrial methylated spirit

Sterile 0.9% saline

This is the correct answer

Surgical spirit

Triclosan

Incorrect answer selected

There is little evidence for superiority when it comes to cleaning agents for wounds.
However, agents containing alcohol should be avoided, such as
Methylated spirit
Surgical spirit
Triclosan and
Alcohol containing chlorhexidine and povidone-iodine solutions.

Hydrogen peroxide is generally not recommended and carries a risk of oxygen embolism.
Tap water
Sterile saline
Aqueous chlorhexidine and
Weak povidone-iodine

solutions are all frequently used to clean wounds in the Emergency department.
Heavily contaminated wounds need thorough lavage, scrubbing and debridement, as
appropriate.

Work Smart

Time taken: 11:02

A 10-year-old boy attends the Emergency department with a scalp laceration he sustained
playing football.
Examination confirms a superficial clean incised wound approximately 2 cm in length.
Which would be the best way of managing this wound?
(Please select 1 option)
Staples
Steristrips
Suture with a non-absorbable suture
Suture with an absorbable suture
Tissue adhesive glue

Correct

Tissue adhesive glue is very useful for scalp wounds particularly in children.
Wounds must be clean and less than about 3 cm in length. Glue should not be used to close
wounds around the eyes or over joints.
It may be performed without the need to use local anaesthesia and may be less distressing
for patients than alternative techniques, such as staples or sutures.
Thus it would be the closure technique of choice in this case.
Steristrips are not appropriate for scalp wounds as hair limits skin adhesion.

Work Smart
Time taken: 11:07

A 54-year-old lady attends the Emergency department with neck pain. She was involved in a
minor road traffic accident two days ago when her car was shunted forward from behind.
On examination there is no bony tenderness and a normal range of movement with no
neurological signs.
Which of the following options is the most appropriate course of action?
(Please select 1 option)

Immobilise the cervical spine immediately

Organise cervical spine x rays

Reassure the patient and prescribe analgesia

This is the correct answer

Reassure the patient and recommend the use of a soft collar

Reassure the patient and suggest bed rest

Incorrect answer selected

Soft tissue injuries to the neck are very common. There is typically a delay in patients
presenting to the emergency department as symptoms worsen following an initial injury.
There should be a low threshold for immobilising the cervical spine and obtaining x rays if
there is:
Cervical spine tenderness
A reduced range of movement or
Any neurological signs.

Patients should be treated with analgesia, preferably non-steroidal anti-inflammatory


preparations.
Collars are not recommended routinely as early mobilisation is the best treatment.
Patients should also be directed to see their own GP for review and appropriate
physiotherapy can be arranged if the symptoms fail to resolve.

Work Smart
Time taken: 11:21

A 29-year-old male attends the Emergency department after stubbing his toe on the end of
his bed. He complains of severe pain in his left second toe.
Examination confirms some mild swelling and pain localised over the distal phalanx of the
left second toe. There is no obvious deformity.
What is the most appropriate course of action?

(Please select 1 option)


Arrange a follow-up appointment in the fracture clinic
Reassure the patient and discharge him
Refer the patient to the on-call team for emergency admission
Request an x ray of the patient's foot
Suggest analgesia, padded buddy strapping and elevation

Correct

Suspected simple, undisplaced fractures of the four small toes do not routinely require an x
ray or routine admission.
They are best managed with analgesia, padded buddy strapping and elevation.
Patients do not usually require follow up and should be advised to return to full activities as
soon as possible

Work Smart
Time taken: 11:27

A 38-year-old female attends the Emergency department after falling onto a muddy path in
the park whilst out running.
She has grazes over both knees and a very dirty deep wound to her left thenar eminence.
An x ray of her hand confirms the presence of a foreign body.
The patient says that she is fully immunised against tetanus.
According to Department of Health guidance which of the following steps should be taken?
(Please select 1 option)
An immediate dose of tetanus immunoglobulin should be given

This is the correct answer

An immediate reinforcing dose of vaccine should be given


Both a reinforcing dose of vaccine and tetanus immunoglobulin should be given immediately
No vaccine or human tetanus immunoglobulin should be given
The patient should be advised to make an appointment with her GP to confirm her immunisation
status Incorrect answer selected

This patient presents with a tetanus-prone wound because of the history of contact with soil
and the fact that there is a foreign body.

Public Health England offers guidance on the Management of patients with tetanus-prone
wounds.
According to this guidance, the fully immunised patient with a tetanus-prone wound should
receive an immediate dose of tetanus immunoglobulin

Work Smart
Time taken: 11:32

A 3-year-old boy is brought to the Emergency department by his father.


The boy is complaining of a painful elbow after being lifted up by his outstretched arms
earlier that day. He is refusing to use his arm.
What is the most likely diagnosis?
(Please select 1 option)
Fracture of the clavicle
Fracture of the lateral epicondyle
Fracture of the medial epicondyle
Fracture of the shaft of the radius/ulnar
Subluxation of the radial head

Correct

Subluxation of the radial head or "pulled elbow" is a common injury in young children. It
results from a direct pull on the arm that causes the radial head to be pulled from its
ligament.
The child usually complains of a painful elbow and there is limited supination and extension.
It is common for children to refuse to use their arm.
If the history is characteristic the diagnosis can be made clinically without the need for x ray.
Treatment includes analgesia and manipulation by supination of the elbow whilst in 90 of
flexion.

Key Learning Points


Cardiology
management of svt

ST elevation without reciprocal changes in context of cardiac syncope suggests


possible LV aneurysm from previous MI.

Biochemistry
Metastatic prostate carcinoma may present with lytic lesions and pathological
fractures

Emergency Medicine
acute respiratory infection
Angio-oedema is an important cause of facial swelling that can cause airway
obstruction.
Abdominal pain
management of svt
Hypoadrenalism is a medical emergency, which often presents with abdominal
pain and a history of weight loss.
Gastrointestinal symptoms
Metastatic prostate carcinoma may present with lytic lesions and pathological
fractures
Pre-operative haematology investigation
Features of acute severe asthma include a PEFR of 33-50% of the predicted
value
The CURB-65 score can be used to help guide a patient's treatment in cases of
community acquired pneumonia.
ST elevation without reciprocal changes in context of cardiac syncope suggests
possible LV aneurysm from previous MI.
Burns
Trauma management
Appropriate management of pre-tibial lacerations and the implication of age and
skin quality.

For routine use 1% lidocaine is the most frequently used anaesthetic for
cleaning, exploring and suturing wounds in the Emergency department.
Adrenaline should not be used in sites supplied by end arteries, such as fingers
and toes.
Wound cleaning preparations.
Most soft tissue injuries to the neck can be managed with simple analgesia.
Recognition of tetanus-prone wounds. Understanding of tetanus immunisation
and the management of tetanus-prone wounds.

ENT
Most soft tissue injuries to the neck can be managed with simple analgesia.
Wound cleaning preparations.
For routine use 1% lidocaine is the most frequently used anaesthetic for
cleaning, exploring and suturing wounds in the Emergency department.
Adrenaline should not be used in sites supplied by end arteries, such as fingers
and toes.
Appropriate management of pre-tibial lacerations and the implication of age and
skin quality.

Gastroenterology
Gastrointestinal symptoms
Abdominal pain

Infectious Diseases
acute respiratory infection

Surgery
Gastrointestinal symptoms
Hypoadrenalism is a medical emergency, which often presents with abdominal
pain and a history of weight loss.
Abdominal pain

Pre-operative haematology investigation


Metastatic prostate carcinoma may present with lytic lesions and pathological
fractures
Appropriate management of pre-tibial lacerations and the implication of age and
skin quality.
For routine use 1% lidocaine is the most frequently used anaesthetic for
cleaning, exploring and suturing wounds in the Emergency department.
Adrenaline should not be used in sites supplied by end arteries, such as fingers
and toes.
Wound cleaning preparations.
Most soft tissue injuries to the neck can be managed with simple analgesia.
Recognition of tetanus-prone wounds. Understanding of tetanus immunisation
and the management of tetanus-prone wounds.

Paediatrics
Angio-oedema is an important cause of facial swelling that can cause airway
obstruction.

Respiratory medicine
Angio-oedema is an important cause of facial swelling that can cause airway
obstruction.
acute respiratory infection

Oncology
Metastatic prostate carcinoma may present with lytic lesions and pathological
fractures

General
Abdominal pain

Anaesthetics & ITU


Pre-operative haematology investigation
management of svt

Trauma
Trauma management

Vous aimerez peut-être aussi